Recherche sur le blog!

Concours Physique ENS Ulm (C/S) M' 1995 (Corrigé)

ULM M’ 95
1.1°) ${A_3} = \rho {A_1} + \tau '{A_2}$ et ${A_4} = \tau {A_1} + \rho '{A_2}$
${I_3} = {A_3}A_3^ * = {\left| \rho \right|^2}{\left| {{A_1}} \right|^2} + {\left| {\tau '} \right|^2}{\left| {{A_2}} \right|^2} + \rho \tau {'^ * }{A_1}A_2^ * + {\rho ^ * }\tau 'A_1^ * {A_2}$
${I_4} = {A_4}A_4^ * = {\left| \tau \right|^2}{\left| {{A_1}} \right|^2} + {\left| {\rho '} \right|^2}{\left| {{A_2}} \right|^2} + \rho {'^ * }\tau \;{A_1}A_2^ * + \rho '{\tau ^ * }A_1^ * {A_2}$
1.2°) Conservation de l’énergie: ${I_3} + {I_4} = {I_1} + {I_2} = ({\left| \rho \right|^2} + {\left| \tau \right|^2}){I_1} + ({\left| {\rho '} \right|^2} + {\left| {\tau '} \right|^2}){I_2} + (\rho \tau {'^*} + \tau \rho {'^*}){A_1}A_2^ * + ({\rho ^ * }\tau ' + {\tau ^ * }\rho ')A_1^ * {A_2}$
${A_2} = 0$$ \Rightarrow $${\left| \rho \right|^2} + {\left| \tau \right|^2} = 1$ et ${A_1} = 0$$ \Rightarrow $${\left| {\rho '} \right|^2} + {\left| {\tau '} \right|^2} = 1$
Il reste $(\rho \tau {'^ * } + \tau \rho {'^ * }){A_1}A_2^ * + ({\rho ^ * }\tau ' + {\tau ^ * }\rho ')A_1^ * {A_2} = 0$ soit ${\mathop{\rm Re}\nolimits} \{ (\rho \tau {'^ * } + \tau \rho {'^ * }){A_1}A_2^ * \} = 0$, valable $\forall $ l’argument de ${A_1}A_2^ * $ d’où $\rho \tau {'^ * } + \tau \rho {'^ * } = 0$
1.3°.α) $\rho = \rho ' = ir$ et $\tau = \tau ' = t$ $ \Rightarrow $${r^2} + {t^2} = 1$ et $irt - irt = 0$
1.3°.β) $\rho = - \rho ' = r$ et $\tau = \tau ' = t$ $ \Rightarrow $${r^2} + {t^2} = 1$ et $rt - rt = 0$
Pour décider de l’un ou l’autre cas, il faut imaginer que la réflexion se fait sur un plan de référence (π) et que le déphasage est du à un parcours dans l’épaisseur d’un dépôt métallique. En plus le plan de référence peut produire un déphasage de 0 ou π mais il est le même que le rayon arrive à gauche ou à droite.
Dans le cas α, le plan de référence est au milieu d’une couche métallique; un déphasage de $\frac{\pi }{2}$ se fait selon le trajet AIB pour le rayon de gauche et selon CID pour le rayon de droite.
Dans le cas β, le plan de référence est confondu avec une des faces de la lame métallique; un déphasage de $\pi $ se fait selon IAB pour le rayon de gauche et aucun déphasage pour le rayon de droite.
Dans le Fabry-Pérot (dépôt métallique sur verre), c’est le cas réalisé.
2.1°) On prend $\rho = - \rho ' = r$ et $\tau = \tau ' = t$ et on pose $\varphi = \frac{{4\pi d}}{\lambda } = \frac{{4\pi \nu d}}{c}$
Les lames sont supposées symétriques l’une de l’autre par rapport au plan médian.
Les amplitudes indiquées dans la lame sont les valeurs à droite pour l’onde se propageant vers la droite et les valeurs à gauche pour l’onde se propageant vers la gauche (sans indication des facteurs exponentiels).
$\frac{{{A_1}}}{{{A_0}}} = \tau \tau '{e^{ - j\frac{\varphi }{2}}}\sum\limits_{q = 0}^\infty {\rho {'^{2q}}{e^{ - jq\varphi }}} $ $ \Rightarrow $ $\frac{{{A_1}}}{{{A_0}}} = \frac{{{t^2}{e^{ - j\frac{\varphi }{2}}}}}{{1 - {r^2}{e^{ - j\varphi }}}}$
$\frac{{{I_1}}}{{{I_0}}} = \frac{{{{(1 - {r^2})}^2}}}{{1 + {r^4} - 2{r^2}\cos \varphi }}$
$\frac{{{I_1}}}{{{I_0}}} = \frac{1}{{1 + m{{\sin }^2}\frac{\varphi }{2}}}$ avec $m = \frac{{4{r^2}}}{{{{(1 - {r^2})}^2}}}$ soit $m = \frac{4}{{{\varepsilon ^2}}}$
$\frac{{{A_2}}}{{{A_0}}} = \rho + \tau \tau '\rho {e^{ - j\varphi }}\sum\limits_{q = 0}^\infty {\rho {'^{2q}}{e^{ - jq\varphi }}} $$ = \rho (1 - \frac{{{t^2}{e^{ - j\varphi }}}}{{1 - {r^2}{e^{ - j\varphi }}}})$$ = \rho \frac{{1 - {e^{ - j\varphi }}}}{{1 - {r^2}{e^{ - j\varphi }}}}$
$\frac{{{I_2}}}{{{I_0}}} = \frac{{m{{\sin }^2}\frac{\varphi }{2}}}{{1 + m{{\sin }^2}\frac{\varphi }{2}}}$ avec $m = \frac{4}{{{\varepsilon ^2}}}$; on bien ${I_1} + {I_2} = {I_0}$ (ainsi, on aurait pu avoir ${I_2}$ à partir de ${I_1}$); la période en $\varphi = \frac{{4\pi \nu d}}{c}$ est égale à $2\pi $ $ \Rightarrow $ ${I_1}$ et ${I_2}$ sont périodiques en fréquence de périodicité $\Delta \nu = \frac{c}{{2d}}$
2.2°) $\varepsilon < < 1 \Rightarrow m = \frac{4}{{{\varepsilon ^2}}} > > 1$.
On a un maximum de ${I_1}$ égal à ${I_0}$ si $\varphi = 2q\pi $ soit si $d = q\frac{c}{{2\nu }}$ et un minimum égal à $\frac{{{I_0}}}{{1 + m}}$ voisin de 0 si $\varphi = (2q + 1)\pi $ soit si $d = (q + \frac{1}{2})\frac{c}{{2\nu }}$; la courbe représentative est donnée ci-contre.
Soit $2\xi $ la largeur à mi-hauteur d’un pic avec $\xi $tel que $d = q\frac{c}{{2\nu }} \pm \xi $ $ \Rightarrow $ $\varphi = 2\pi q \pm \frac{{4\pi \nu \xi }}{c}$ et ${I_1} = \frac{{{I_0}}}{2}$ $ \Rightarrow $ $m{\sin ^2}(\pi q \pm \frac{{2\pi \nu \xi }}{c}) = 1$ $ \Rightarrow $ $\sin (\frac{{2\pi \nu \xi }}{c}) = \pm \frac{1}{{\sqrt m }}$ et donc $2\xi = \frac{c}{{\pi \nu \sqrt m }} < < \frac{c}{{2\nu }}$ $ \Rightarrow $ les pics sont très étroits.
2.3°) Entre deux pics successifs $\Delta d = \frac{c}{{2\nu }} = \frac{\lambda }{2}$ (la moitié de la longueur d’onde).
2.4°) Si $\nu \to \nu + d\nu $, les abscisses des pics associés à $\nu $ sont ${d_q} = q\frac{c}{{2\nu }}$ et celles des pics associés à $\nu + d\nu $ sont $d{'_q} = q\frac{c}{{2(\nu + d\nu )}}$; on obtient une série de doubles pics.
2.5°) A la limite de séparation le bord droit, à mi-hauteur, d’un pic, coïncide avec le bord gauche, à mi-hauteur de l’autre pic.
La courbe ci-contre représente les deux pics dans cette position et la courbe résultante.
On aura séparation si $d{'_q} - {d_q} > 2\xi $ soit si $q\frac{c}{2}(\frac{1}{\nu } - \frac{1}{{\nu + d\nu }}) > 2\xi = \frac{2}{{\pi \nu \sqrt m }}$ $ \Rightarrow $ $\delta \nu > \frac{{2\nu }}{{q\pi \sqrt m }}$
Mais $d = \frac{{qc}}{{2\nu }}$ $ \Rightarrow $ $\delta \nu = \frac{c}{{d\pi \sqrt m }}$ ou $\delta {\nu _m} = \frac{{\varepsilon c}}{{2\pi d}}$
2.6°) $\varepsilon = {10^{ - 3}}\;et\;m = {4.10^6}$ $ \Rightarrow $ $\delta {\nu _m} = {10^6}Hz$
$d = q\frac{c}{{2\nu }}$ $ \Rightarrow $ $\Delta d = - d\frac{{\Delta \nu }}{\nu }$ or $\nu = \frac{c}{\lambda } = {4.74.10^{14}}Hz$
soit $\Delta d = - {10^{ - 10}}m$
2.7°) Le double pic peut être dû au voisinage d’un pic d’ordre q pour la radiation de longueur d’onde $\lambda $et d’un pic d’ordre q+p pour la radiation de longueur d’onde $\lambda + d\lambda $: ${d_\lambda } = q\frac{\lambda }{2}$ et ${d_{\lambda + d\lambda }} = (q + p)\frac{{\lambda + d\lambda }}{2}$; on a confusion des deux pics si ${d_\lambda } = {d_{\lambda + d\lambda }}$ $ \Rightarrow \;d = - p\frac{{{\lambda ^2}}}{{2\Delta \lambda }}$ avec $p = \pm 1, \pm 2,...$; pour $\Delta \nu $ de l’ordre de $\Delta {\nu _m}$, $d = - p\frac{c}{{\Delta {\nu _m}}} = - 300p$ soit 300 m pour $p = \pm 1$; or d=5cm; pour des raies de structure hyperfine:
on peut conclure avec certitude.
2.8°) ${I_{1\;\max }} = {I_0}$
2.9°) ${I_1} + {I_2} = {I_0}$
2.10°) $\frac{{{A_ - }}}{{{A_0}}} = \rho '\tau {e^{ - j\frac{\varphi }{2}}}\sum\limits_{q = 0}^\infty {\rho {'^{2q}}{e^{ - jq\varphi }}} $=$\frac{{\rho '\tau {e^{ - j\frac{\varphi }{2}}}}}{{1 - \rho {'^2}{e^{ - j\varphi }}}}$ $ \Rightarrow $ $\frac{{{I_ - }}}{{{I_0}}} = \frac{{1 - \varepsilon }}{\varepsilon }\frac{1}{{1 + m{{\sin }^2}\frac{\varphi }{2}}}$
$\frac{{{A_ - }}}{{{A_0}}} = \tau \sum\limits_{q = 0}^\infty {\rho {'^{2q}}{e^{ - jq\varphi }}} $=$\frac{\tau }{{1 - \rho {'^2}{e^{ - j\varphi }}}}$ $ \Rightarrow $ $\frac{{{I_ - }}}{{{I_0}}} = \frac{1}{\varepsilon }\;\frac{1}{{1 + m{{\sin }^2}\frac{\varphi }{2}}}$
$\varepsilon < < 1$ ${I_ - } \approx {I_ + } \approx \;\frac{1}{\varepsilon }\;\frac{1}{{1 + m{{\sin }^2}\frac{\varphi }{2}}}$ avec $m = \frac{4}{{{\varepsilon ^2}}}$; il y a résonance pour $I = {I_0}$.
2.11°) A travers une surface S parallèle aux miroirs, entre eux, le flux du vecteur de Poynting est $\vec \pi .\vec S = ({\vec \pi _ + } - {\vec \pi _ - }).\vec S$ et représente le différence entre le flux d’énergie dû à l’onde + et celui dû à l’onde -.
2.12°) Avec n entier relatif: $\varphi = 2\pi n + \frac{{4\pi d}}{c}(\nu - {\nu _n})$; à mi-hauteur $\sin 2\pi \frac{d}{c}(\nu - {\nu _n}) = \pm \frac{1}{{\sqrt m }}$ d’où $\delta \nu = \frac{c}{{d\pi \sqrt m }}$ et $\frac{{{I_1}}}{{{I_0}}} = \frac{1}{{1 + 4{{\left( {\frac{{\nu - {\nu _m}}}{{\Delta \nu }}} \right)}^2}}}$.
Le facteur 4 a été oublié dans le texte qui précisait bien largeur à mi-hauteur et non demi-largeur à mi-hauteur.
$Q = \frac{{{\nu _n}}}{{\Delta \nu }} = \frac{{d\pi \sqrt m \nu }}{c} = \frac{{2d\pi \nu }}{{\varepsilon c}} = \frac{{2d\pi }}{{\varepsilon \lambda }}$ $Q = {5.10^8}$
2.13°) On tient compte des durées de propagation; on étudie le régime transitoire du Fabry-Pérot; on prend la date t=0 quand le premier rayon transmis sort de l’appareil.
$\frac{{{A_1}}}{{{A_0}}} = {t^2}\sum\limits_{l = 0}^n {{r^{2l}}} {e^{ - jl\varphi }}$ avec $\varphi = 2q\pi $ (résonance) soit $\frac{{{A_1}}}{{{A_0}}} = {t^2}\sum\limits_{l = 0}^n {{r^{2l}}} $$ = 1 - {r^{2(n + 1)}}$ et $\frac{{{I_1}}}{{{I_0}}} = {\left[ {1 - {r^{2(n + 1)}}} \right]^2}$ avec $n = \frac{{ct}}{{2d}}$; ${I_1}$ tend vers zéro quand n tend vers l’infini; on se ramène à $\frac{{{I_1}}}{{{I_0}}} = {\left[ {1 - {r^{\frac{{ct}}{d}}}} \right]^2}$ après un décalage des temps légitime de 2d/c quand on passe d’une fonction discontinue en escalier à une fonction continue.
$\frac{{{A_1}}}{{{A_0}}} = {t^2}\sum\limits_{l = 0}^n {{r^{2l}}} {e^{ - jl\varphi }}$$ = {r^{2n + 1}}$ et $\frac{{{I_2}}}{{{I_0}}} = {r^{2(2n + 1)}}$; ${I_2}$ tend vers zéro quand n tend vers l’infini.
2.14°) ${r^2} = 1 - \varepsilon \approx {e^{ - \varepsilon }}$ $ \Rightarrow \;{I_1}(t) = {I_0}{\left( {1 - {e^{ - \frac{{\varepsilon ct}}{{2d}}}}} \right)^2}$; on ne trouve pas la formule du texte dans lequel l’exposant 2 a vraisemblablement été oublié.
2.15°) ${\tau _c} = \frac{{2d}}{{c\varepsilon }}$
2.16°) ${\tau _c}\Delta \nu = \frac{1}{\pi }$ (on trouve ${\tau _c}\Delta \nu = \frac{1}{{2\pi }}$ en prenant la demi-largeur à demi-hauteur).
2.17°) A l’instant initial, l’onde arrête d’émerger du Fabry-Pérot; la série géométrique est tronquée au début alors que c’était à la fin en 2.13°; $\frac{{{A_1}}}{{{A_0}}} = {\left( {\frac{{{A_1}}}{{{A_0}}}} \right)_{n \to \infty (2.13^\circ )}} - {\left( {\frac{{{A_1}}}{{{A_0}}}} \right)_{(2.13^\circ )}} = {r^{2(n + 1)}}$d’où $\frac{{{I_1}}}{{{I_0}}} = {r^{4(n + 1)}}$ et $\frac{{{I_1}}}{{{I_0}}} = {e^{ - \frac{{\varepsilon ct}}{d}}}$
et de même: $\frac{{{A_2}}}{{{A_0}}} = {\left( {\frac{{{A_2}}}{{{A_0}}}} \right)_{n \to \infty (2.13^\circ )}} - {\left( {\frac{{{A_2}}}{{{A_0}}}} \right)_{(2.13^\circ )}} = - {r^{2n + 1}}$d’où $\frac{{{I_2}}}{{{I_0}}} = {r^{2(2n + 1)}}$ et $\frac{{{I_2}}}{{{I_0}}} = {e^{ - \frac{{\varepsilon ct}}{d}}}$
2.18°) Pour les deux intensités, la limite, quand n tend vers l’infini, est nulle alors qu’en 2.13° elle est ${I_0}$ et 0 respectivement.
2.19°) L’onde n’est pas résonnante: si $n \to \infty $, pour l’établissement: $\frac{{{I_1}}}{{{I_0}}} \to \frac{1}{{1 + m{{\sin }^2}\frac{\varphi }{2}}}$ et $\frac{{{I_2}}}{{{I_0}}} \to \frac{{m{{\sin }^2}\frac{\varphi }{2}}}{{1 + m{{\sin }^2}\frac{\varphi }{2}}}$ alors que pour la rupture, les deux intensités tendent vers zéro.
2.20°) On indique « lentement » car il faut qu’un régime quasi-stationnaire soit réalisé et donc que la durée de déplacement soit très supérieure au temps de stockage de la cavité.
3.1°) On considère 2 miroirs identiques (pas dit dans le texte): $A\xrightarrow[{{M}_{1}}]{}{{A}_{1}}\xrightarrow[{{M}_{2}}]{}{{A}_{2}}\xrightarrow[{{M}_{1}}]{}{{A}_{3}}\xrightarrow[{{M}_{2}}]{}{{A}_{4}}$
${\overline {FA} _1}.\overline {FA} = {\overline {FA} _1}.{\overline {FA} _2}$ (formule de conjugaison de Newton) implique qu’après deux réflexions l’image A2 se confond avec l’objet A; le grandissement transversal, selon les formules de Newton, après ces deux réflexions est ${\gamma _t} = \frac{{ - {f_2}}}{{\overline {F{A_1}} }}.\frac{{ - \overline {F{A_1}} }}{{{f_1}}} = - 1$ et l’image d’un petit objet rectiligne perpendiculaire à l’axe est renversée; après 4 réflexions, deux sur chaque miroir, l’image d’un petit objet rectiligne perpendiculaire à l’axe se confond avec l’objet. D’après la relation de Lagrange-Helmoltz $G{\gamma _t} = - 1$ pour un miroir et +1 pour un nombre pair de miroirs; on a donc le grandissement angulaire G=1.
Après 4 réflexions le rayon repart du même point de l’axe dans les mêmes direction et sens.
3.2°) L = 2f = R (rayon des miroirs)
Soit A le point sur l’axe de révolution; tout rayon issu de A repasse en A après 4 réflexions, dans le cadre de l’approximation de Gauss, d’où: $(A{A_4}) = 4L = 8f = 4R$; le chemin optique ne dépend pas du rayon considéré (stigmatisme approché); il ne dépend pas de l’angle d’inclinaison; en posant $\varphi = \frac{{8\pi \nu R}}{c}$, on a le tableau suivant des amplitudes internes (juste avant sortie de la cavité):
Faisceau 1 Faisceau 2 Faisceau 3 Faisceau 4
Passage 1 ${A_0}\tau {e^{ - j{\varphi _0}}}$ ${A_0}\tau \rho '{e^{ - j{\varphi _1}}}$ ${A_0}\tau \rho {'^2}{e^{ - j{\varphi _2}}}$ ${A_0}\tau \rho {'^3}{e^{ - j\varphi }}$
Passage 2 ${A_0}\tau \rho {'^4}{e^{ - j{\varphi _0}}}{e^{ - j\varphi }}$ ${A_0}\tau \rho {'^5}{e^{ - j{\varphi _1}}}{e^{ - j\varphi }}$ ${A_0}\tau \rho {'^6}{e^{ - j{\varphi _2}}}{e^{ - j\varphi }}$ ${A_0}\tau \rho {'^7}{e^{ - 2j\varphi }}$
Passage 3 ${A_0}\tau \rho {'^8}{e^{ - j{\varphi _0}}}{e^{ - 2j\varphi }}$ ${A_0}\tau \rho {'^9}{e^{ - j{\varphi _1}}}{e^{ - 2j\varphi }}$ ${A_0}\tau \rho {'^{10}}{e^{ - j{\varphi _2}}}{e^{ - 2j\varphi }}$ ${A_0}\tau \rho {'^{11}}{e^{ - 3j\varphi }}$
où ${\varphi _0},{\varphi _1},{\varphi _2},\varphi $ sont les déphasages le long des trajets internes.
$\frac{{{A_1}}}{{{A_0}}} = \tau \tau '{e^{ - j{\varphi _0}}}\sum\limits_{q = 0}^\infty \rho {'^{4q}}{e^{ - jq\varphi }} = \frac{{{t^2}{e^{ - j{\varphi _0}}}}}{{1 - {r^4}{e^{ - j\varphi }}}}$ $ \Rightarrow $ $\frac{{{I_1}}}{{{I_0}}} = \frac{{{{(1 - {r^2})}^2}}}{{{{(1 - {r^4})}^2} + 4{r^4}{{\sin }^2}\frac{\varphi }{2}}}$
$\frac{{{A_2}}}{{{A_0}}} = \tau \tau '\rho '{e^{ - j{\varphi _1}}}\sum\limits_{q = 0}^\infty \rho {'^{4q}}{e^{ - jq\varphi }} = \frac{{ - r{t^2}{e^{ - j{\varphi _1}}}}}{{1 - {r^4}{e^{ - j\varphi }}}}$ $ \Rightarrow $ $\frac{{{I_2}}}{{{I_0}}} = \frac{{{r^2}{{(1 - {r^2})}^2}}}{{{{(1 - {r^4})}^2} + 4{r^4}{{\sin }^2}\frac{\varphi }{2}}}$
$\frac{{{A_3}}}{{{A_0}}} = \tau \tau '\rho {'^2}{e^{ - j{\varphi _2}}}\sum\limits_{q = 0}^\infty \rho {'^{4q}}{e^{ - jq\varphi }} = \frac{{{r^2}{t^2}{e^{ - j{\varphi _2}}}}}{{1 - {r^4}{e^{ - j\varphi }}}}$ $ \Rightarrow $ $\frac{{{I_3}}}{{{I_0}}} = \frac{{{r^4}{{(1 - {r^2})}^2}}}{{{{(1 - {r^4})}^2} + 4{r^4}{{\sin }^2}\frac{\varphi }{2}}}$
$\frac{{{A_4}}}{{{A_0}}} = \rho + \tau \tau '\rho {'^3}{e^{ - j\varphi }}\sum\limits_{q = 0}^\infty \rho {'^{4q}}{e^{ - jq\varphi }} = \frac{{r(1 - {r^2}{e^{ - j\varphi }})}}{{1 - {r^4}{e^{ - j\varphi }}}}$ $ \Rightarrow $ $\frac{{{I_4}}}{{{I_0}}} = \frac{{{r^2}\left[ {{{(1 - {r^2})}^2} + 4{r^2}{{\sin }^2}\frac{\varphi }{2}} \right]}}{{{{(1 - {r^4})}^2} + 4{r^4}{{\sin }^2}\frac{\varphi }{2}}}$
On vérifie que la somme des quatre intensités est l’intensité incidente.
3.3°) Le trajet admet z’z comme axe de symétrie d’où ${\varphi _2} - {\varphi _0} = \frac{\varphi }{2}$ et $\frac{{{A_3}}}{{{A_1}}} = {r^2}{e^{ - j\frac{\varphi }{2}}}$; à la résonance $\varphi = 2q\pi $ $ \Rightarrow $
${A_1}\;et\;{A_3}$ sont en phase si q est pair et en opposition de phase si q est impair.
3.4°) Les rayons (1) et (3) sont confondus avec z’z; on se retrouve dans le cas d’un Fabry-Pérot plan-plan d’où $\frac{{{I_{13}}}}{{{I_0}}} = \frac{{{{(1 - {r^2})}^2}}}{{1 + {r^4} - 2{r^2}\cos \frac{\varphi }{2}}}$ ce qu’on retrouve mais par un calcul compliqué en faisant interférer les ondes (1) et (3).
4.1°) Du fait de la symétrie de révolution (xOz et yOz, plans de symétrie équivalents), on peut montrer facilement qu’entre grandeurs de sortie et grandeurs d’entrée, pour un rayon quelconque: $\left( {\begin{array}{*{20}{c}}{{x_{n + 1}}}\\{{\alpha _{n + 1}}}\\{{y_{n + 1}}}\\{{\beta _{n + 1}}}\end{array}} \right) = \left( {\begin{array}{*{20}{c}}a&b&0&0\\c&d&0&0\\0&0&a&b\\0&0&c&d\end{array}} \right)\left( {\begin{array}{*{20}{c}}{{x_n}}\\{{\alpha _n}}\\{{y_n}}\\{{\beta _n}}\end{array}} \right)$
Ceci revient à étudier la projection d’un rayon sur les plans xOz et yOz; il suffit d’étudier la marche d’un rayon méridien.
4.2°) On obtient quatre relations:
a) Dans le vide de gauche à droite: $y{'_n} = {y_n} + {\alpha _n}L$
b) Réflexion sur M2 , à droite: avec la relation de conjugaison (origine au sommet): $\alpha {'_n} = - {\alpha _n} - \frac{{2y{'_n}}}{{\overline {{S_2}{C_2}} }}$
c) Dans le vide de droite à gauche: ${y_{n + 1}} = {y_n} - \alpha {'_n}L$
d) Réflexion sur le miroir de gauche M1: ${\alpha _{n + 1}} = - \alpha {'_n} - \frac{{2{y_{n + 1}}}}{{\overline {{S_1}{C_1}} }}$
D’où par éliminations successives, la relation matricielle: $\left( {\begin{array}{*{20}{c}}{{y_{n + 1}}}\\{{\alpha _{n + 1}}}\end{array}} \right) = \left( {\begin{array}{*{20}{c}}{1 + \frac{{2L}}{{\overline {{S_2}{C_2}} }}}&{2L(1 + \frac{L}{{\overline {{S_2}{C_2}} }})}\\{\frac{2}{{\overline {{S_2}{C_2}} }} - \frac{2}{{\overline {{S_1}{C_1}} }} - \frac{{4L}}{{\overline {{S_1}{C_1}\;} \overline {{S_2}{C_2}} }}}&{1 + \frac{{2L}}{{\overline {{S_2}{C_2}} }} - \frac{{4L}}{{\overline {{S_1}{C_1}} }}(1 + \frac{L}{{\overline {{S_2}{C_2}} }})}\end{array}} \right)\left( {\begin{array}{*{20}{c}}{{y_n}}\\{{\alpha _n}}\end{array}} \right)$
On remarque que le déterminant de la matrice de transfert vaut l’unité; on peut le voir par calcul direct ou bien en calculant la matrice comme produit de quatre matrices élémentaires chacune de déterminant unité.
4.3°) $\left( {\begin{array}{*{20}{c}}{{y_n}}\\{{\alpha _n}}\end{array}} \right) = {M^n}\left( {\begin{array}{*{20}{c}}{{y_0}}\\{{\alpha _0}}\end{array}} \right)$; dans la base des vecteurs propres de M, M et sa puissance n sont diagonales d’éléments diagonaux les valeurs propres: $({\lambda _1},{\lambda _2})$ et $(\lambda _1^n,\lambda _2^n)$; on en déduit que la solution dans la base des vecteurs propres est ${Y_n} = \lambda _1^n{Y_0}$ et ${A_n} = \lambda _2^n{A_0}$; à l’aide de la matrice de changement de base, on repasse dans la base initiale; ${y_n}$ et ${\alpha _n}$ sont donc combinaisons linéaires de $\lambda _1^n\;et\;\lambda _2^n$; les coefficients ne dépendent pas de n; l’équation aux valeurs propres, avec le déterminant de la matrice de transfert égal à l’unité est: ${\lambda ^2} - \lambda trace(M) + 1 = 0$; on en déduit que le produit des valeurs propres (complexes) vaut l’unité: ${\lambda _1}{\lambda _2} = 1$; les valeurs propres peuvent s’écrire sous la forme: ${\lambda _1} = \rho {e^{i\theta }}$ et ${\lambda _2} = \frac{1}{\rho }{e^{ - i\theta }}$; si $\rho \ne 1$ l’une des racines a un module supérieur à l’unité et donc ${y_n}$ et ${\alpha _n}$ divergent quand n devient grand: il y a instabilité. Pour avoir stabilité, il faut $\rho = 1$.
4.4°) Les deux valeurs propres sont imaginaires: ${\lambda _1} = {e^{i\theta }}$ et ${\lambda _2} = {e^{ - i\theta }}$; de ce fait: $trace(M) = a + d = 2\cos \theta $; la condition de stabilité est donc: $ - 2 \le a + d \le 2$ soit en remplaçant a et d par leurs valeurs et en posant ${g_i} = 1 - \frac{L}{{{R_i}}}$ le résultat: $0 \le {g_1}{g_2} \le 1$.
4.5°) Le domaine de stabilité est compris entre les deux branches de l’hyperbole équilatère.
Pour le Fabry-Pérot plan-plan les rayons de courbures sont infinis: le point (1,1) est le point représentatif.
Pour le Fabry-Pérot confocal les rayons sont égaux à L, le point représentatif est l’origine des axes.
4.6°) On veut ${y_n} = {y_0}$, ${x_n} = {x_0}$, ${\alpha _n} = {\alpha _0}$, ${\beta _n} = {\beta _0}$ $ \Rightarrow {M^n} = I$ (matrice unité); dans la base des vecteurs propres, cette propriété reste vraie d’où $\lambda _1^n = 1\;et\;\lambda _2^n = 1$ et du fait que le produit des valeurs propres vaut l’unité: ${\lambda _1} = {e^{\frac{{2\pi iq}}{n}}}$ et ${\lambda _1} = {e^{ - \frac{{2\pi iq}}{n}}}$; d’où ${\lambda _1} + {\lambda _2} = 2\cos \frac{{2\pi q}}{n} = a + d$; avec les rayons égaux $\cos \frac{{2\pi q}}{n} = 1 - \frac{{4L}}{R} + 2\frac{{{L^2}}}{{{R^2}}}$ ce qui conduit à l’équation ${\left( {\frac{L}{R}} \right)^2} - 2\left( {\frac{L}{R}} \right) + {\sin ^2}\frac{{\pi q}}{n} = 0$ de racines $\frac{L}{R} = 1 \pm \cos \frac{{q\pi }}{n}$ $q \in \left\{ {0..n - 1} \right\}$.
4.7°) ${x_k}\;et\;{y_k}$ sont combinaisons linéaires de $\lambda _1^k\;et\;\lambda _2^k$ avec des coefficients indépendants de k; on en déduit ${x_k} = A\cos (2\pi \frac{{kq}}{n} + {\varphi _x})$ et ${y_k} = A\cos (2\pi \frac{{kq}}{n} + {\varphi _y})$; il suffit que $qk \in \left\{ {0..n - 1} \right\}$ pour avoir toutes les valeurs possibles; on a bien n taches qui se répartissent sur une ellipse définie par son équation paramétrique.
4.8°) Si le trou est assez petit, le faisceau ne ressort plus quand L n’a pas exactement une des valeurs précédentes mais les taches en grand nombre, recouvrent une courbe proche de l’ellipse.
4.9°) Cavité multipassage: si n est élevé, le faisceau sera très affaibli (multiplication de l’intensité par ${r^{2n}}$); pour le Fabry-Pérot sphérique, à résonance, ${I_4} = \frac{{{I_0}}}{4}$; il est meilleur du point de vue de l’intensité émergente.
5.1°) $\Delta \vec E - \frac{1}{{{c^2}}}\frac{{{\partial ^2}\vec E}}{{\partial {t^2}}} = 0$ et $\Delta \vec B - \frac{1}{{{c^2}}}\frac{{{\partial ^2}\vec B}}{{\partial {t^2}}} = 0$
5.2°) Pour un métal parfait, en son voisinage: $\vec E = \frac{\sigma }{{{\varepsilon _0}}}\vec n$ et $\vec B = {\mu _0}{\vec j_s} \wedge \vec n$
5.3°) $div\vec E = 0 \Rightarrow \frac{{\partial f}}{{\partial y}} = 0$ soit $f = f(x,z)$
5.4°) $\frac{{{\partial ^2}f}}{{\partial {x^2}}} + \frac{{{\partial ^2}f}}{{\partial {z^2}}} + \frac{{{\omega ^2}}}{{{c^2}}}f = 0$
5.5°) Méthode de séparation des variables:$f(x,z)=h(x)k(z)\Rightarrow \frac{h''}{h}+\frac{k''}{k}+\frac{{{\omega }^{2}}}{{{c}^{2}}}=0$; pour pouvoir assurer les conditions limites, il faut: $\frac{h''}{h}=-{{\alpha }^{2}}$, $\frac{k''}{k}=-{{\gamma }^{2}}$ avec ${\alpha ^2} + {\gamma ^2} = \frac{{{\omega ^2}}}{{{c^2}}}$; les solutions sont sinusoïdales
et données par $h = A\cos \alpha x + B\sin \alpha x$ et $k = A\cos \gamma z + B\sin \gamma z$;
Pour $x = \pm \frac{X}{2}$ le champ électrique doit être normal $ \Rightarrow \vec E = 0$ et $h( \pm X/2) = 0$ $ \Rightarrow $ $A\cos \frac{{\alpha X}}{2} + B\sin \frac{{\alpha X}}{2} = A\cos \frac{{\alpha X}}{2} - B\sin \frac{{\alpha X}}{2} = 0$ et donc $\alpha = {n_1}\frac{\pi }{X}$ ${n_1} \in \aleph $
Pour $z = \pm \frac{Z}{2}$ le champ électrique doit être normal $ \Rightarrow \vec E = 0$ et $k( \pm Z/2) = 0$ $ \Rightarrow $
$C\cos \frac{{\gamma Z}}{2} + D\sin \frac{{\gamma Z}}{2} = C\cos \frac{{\gamma Z}}{2} - D\sin \frac{{\gamma Z}}{2} = 0$ et donc $\gamma = {n_2}\frac{\pi }{Z}$ ${n_2} \in \aleph $
Selon la parité de ces entiers: $f(x,z) = {E_0}\left( {\begin{array}{*{20}{c}}{\cos \alpha x}\\{ou}\\{\sin \alpha x}\end{array}} \right)x\left( {\begin{array}{*{20}{c}}{\cos \gamma z}\\{ou}\\{\sin \gamma z}\end{array}} \right)$; la solution du texte n’est valable que si les deux entiers sont impairs.
5.6°) $\alpha = {n_1}\frac{\pi }{X}$ $\gamma = {n_2}\frac{\pi }{Z}$
5.7°) $\frac{{{\omega ^2}}}{{{c^2}}} = \left( {\frac{{n_1^2}}{{{X^2}}} + \frac{{n_2^2}}{{{Z^2}}}} \right){\pi ^2}$ ${n_1}\;et\;{n_2} \in \aleph $
5.8°) On a une condition de résonance analogue à $d = q\frac{c}{{2\nu }}$
5.9°) $f(z) = {E_0}\cos \frac{{n\pi z}}{Z}$ si n est impair ou $f(z) = {E_0}\sin \frac{{n\pi z}}{Z}$ si n est pair; $\vec E = {E_0}\cos \frac{{n\pi z}}{Z}{\mathop{\rm Re}\nolimits} ({e^{i\omega t}}){\vec e_y}$ si n est impair.
5.10°) $\vec j = \sigma \vec E$; on fait l’hypothèse que la conductivité est réelle; l’équation de Maxwell-Ampère en régime sinusoïdal forcé s’écrit: $\vec \nabla \wedge \vec B = {\mu _0}(\sigma + i{\varepsilon _0}\omega )\vec E$ et donc $\vec \nabla \wedge \vec B \approx {\mu _0}\sigma \vec E$ si $\sigma > > {\varepsilon _0}\omega $; d’après l’équation de Maxwell-Faraday $\vec \nabla \wedge \vec E = - \frac{{\partial \vec B}}{{\partial t}}$ et la formule de calcul vectoriel $\vec \nabla \wedge (\vec \nabla \wedge \vec E) = \vec \nabla (\vec \nabla .\vec E) - \vec \Delta \vec E$, on obtient l’équation $\Delta \vec E = {\mu _0}\sigma \frac{{\partial \vec E}}{{\partial t}}$.
5.11°) $\vec E = \vec e(z){e^{i\omega t}}$ $ \Rightarrow $ $\frac{{{\partial ^2}\vec e}}{{\partial {z^2}}} = \frac{{2i}}{{{\delta ^2}}}\vec e$ avec ${\delta ^2} = \frac{2}{{{\mu _0}\sigma \omega }}$ et $\vec e = \vec K{e^{ \pm \frac{{1 + i}}{\delta }z}}$
La solution générale est combinaison linéaire des deux solutions: $\vec{E}=\vec{E}{{'}_{0}}{{e}^{-\frac{1+i}{\delta }z}}{{e}^{i\omega t}}+\vec{E}{{''}_{0}}{{e}^{\frac{1+i}{\delta }z}}{{e}^{i\omega t}}$ soit
$\vec{E}=\vec{E}{{'}_{0}}{{e}^{-\frac{z}{\delta }}}{{e}^{i(\omega t-\frac{z}{\delta })}}+\vec{E}{{''}_{0}}{{e}^{+\frac{z}{\delta }}}{{e}^{i(\omega t+\frac{z}{\delta })}}$. Pour une propagation selon Oz: $\vec E = \vec E{'_0}{e^{ - \frac{z}{\delta }}}{e^{i(\omega t - \frac{z}{\delta })}}$; en plus le champ est transverse.
5.12°) Dans le métal:
$\vec \nabla \wedge \vec E = - i\omega \vec B$$ \Rightarrow $$\vec B = \frac{i}{\omega }\left( {\frac{{1 + i}}{\delta }} \right)({E_{0y}}{\vec e_x} - {E_{0x}}{\vec e_y}){e^{ - \frac{{1 + i}}{\delta }z}}{e^{i\omega t}}$ ou $\vec B = \frac{{i - 1}}{{\omega \delta }}(\vec E{'_0} \wedge {\vec e_z}){e^{ - \frac{{1 + i}}{\delta }z}}{e^{i\omega t}}$
Dans le vide:
$\vec E = {E_0}\cos \alpha z{e^{i\omega t}}{\vec e_y}$ et $\vec B = \frac{1}{{i\omega }}\frac{{\partial E}}{{\partial z}}{\vec e_x}$ $ \Rightarrow $ $\vec B = i\frac{{{E_0}\alpha }}{\omega }\sin \alpha z\;{e^{i\omega t}}{\vec e_x}$; or $\alpha = \frac{\omega }{c}$ est la norme du vecteur d’onde $ \Rightarrow $ $\vec B = i\frac{{{E_0}}}{c}\sin \alpha z\;{e^{i\omega t}}{\vec e_x}$
5.13°) En z=Z/2, on écrit:
la continuité du champ électrique selon y’y: $E{'_0}{e^{ - (1 + i)\frac{z}{{2\delta }}}} = {E_0}\cos \alpha \frac{Z}{2}$
la continuité du champ magnétique (pas de courants surfaciques) selon x’x: $\alpha {E_0}\sin \alpha \frac{Z}{2} = \frac{{(i - 1)}}{{\delta \omega }}E{'_0}{e^{ - \frac{{(1 + i)}}{{2\delta }}Z}}$
Pour avoir des champs non nuls, il faut que le déterminant du système soit nul: $\tan \alpha \frac{Z}{2} = \frac{{1 + i}}{{\delta \alpha }}$.
5.14°) Avec les expressions de $\delta $et de $\alpha $ on a $\tan \omega \frac{Z}{{2c}} = (1 + i)\sqrt {\frac{\sigma }{{2{\varepsilon _0}\omega }}} $ et ${\omega _0} = p\pi \frac{c}{Z} = (2q + 1)\pi \frac{c}{Z}$ (p est impair car le champ électrique est en cosinus); on a $\omega = {\omega _0} + \varepsilon $ $ \Rightarrow $ $\tan \left( {(q + \frac{1}{2})\pi + \varepsilon \frac{Z}{{2c}}} \right) = (1 + i){\left( {\frac{\sigma }{{2{\varepsilon _0}\omega }}} \right)^{\frac{1}{2}}}$
Au premier ordre près, on peut remplacer $\omega $ par ${\omega _0}$ et du fait que deux angles complémentaires ont des tangentes dont le produit vaut 1, on a: $ - \frac{1}{{\tan \frac{{\varepsilon Z}}{{2c}}}} \approx (1 + i)\sqrt {\frac{\sigma }{{2{\varepsilon _0}{\omega _0}}}} $; en assimilant la tangente à l’angle et en remplaçant ${\omega _0}$ par sa valeur, on obtient la formule du texte: $\omega = \frac{c}{Z}\left( {p\pi - \sqrt {\frac{{2{\varepsilon _0}{\omega _0}}}{\sigma }} + i\sqrt {\frac{{2{\varepsilon _0}{\omega _0}}}{\sigma }} } \right)$
5.15°) ${e^{i\omega t}} = {e^{i\frac{c}{Z}\left( {p\pi - \sqrt {\frac{{2{\varepsilon _0}{\omega _0}}}{\sigma }} } \right)t}}{e^{ - \sqrt {\frac{{2{\varepsilon _0}{\omega _0}}}{\sigma }} \frac{c}{Z}t}}$ $ \Rightarrow $ amortissement temporel de constante de temps $\tau = \frac{Z}{c}\sqrt {\frac{\sigma }{{2{\varepsilon _0}{\omega _0}}}} $; c’est comparable au régime transitoire du Fabry-Pérot quand on enlève la source (2.17°).
5.16°) $\vec \pi = \frac{{\vec E \wedge \vec B}}{{{\mu _0}}}$$ = \frac{{(\vec E + {{\vec E}^*}) \wedge (\vec B + {{\vec B}^*})}}{{4{\mu _0}}}$$ = \frac{{(\vec E \wedge \vec B + {{\vec E}^*} \wedge {{\vec B}^*} + {{\vec E}^*} \wedge \vec B + \vec E \wedge {{\vec B}^*})}}{{4{\mu _0}}}$$ \Rightarrow $
$\left\langle {\vec \pi } \right\rangle = $$\frac{{(\vec E \wedge {{\vec B}^*} + {{\vec E}^*} \wedge \vec B)}}{{4{\mu _0}}}$=$\frac{{{\mathop{\rm Re}\nolimits} ({{\vec E}^*} \wedge \vec B)}}{{2{\mu _0}}}$ ; or $\vec E(Z/2,t) = {E_0}\cos \alpha \frac{Z}{2}\;{e^{i\omega t}}{\vec e_y}$ et $\vec B(Z/2,t) = i\frac{{{E_0}}}{c}\sin \alpha \frac{Z}{2}\;{e^{i\omega t}}{\vec e_x}$ $ \Rightarrow $ $\left\langle {\vec \pi } \right\rangle = $-$\frac{{E_0^2}}{{2{\mu _0}c}}{\mathop{\rm Re}\nolimits} [i{(\cos \alpha \frac{Z}{2})^*}\sin \alpha \frac{Z}{2}]{\vec e_z}$ puis 5.13°$ \Rightarrow $$\left\langle {\vec \pi } \right\rangle = $-$\frac{{E_0^2}}{{2{\mu _0}c}}{\mathop{\rm Re}\nolimits} \left[ {\frac{{i(1 + i)}}{{\delta \alpha }}\;{{\left| {\cos \alpha \frac{Z}{2}} \right|}^2}} \right]{\vec e_z}$ = $\frac{{E_0^2}}{{2{\mu _0}}}\frac{{{{\left| {\cos \alpha \frac{Z}{2}} \right|}^2}}}{{\delta \omega }}\;{\vec e_z}$
On remplace $\alpha = \frac{\omega }{c}$ et $\omega $ par leurs valeurs ce qui donne:
$\left\langle {\vec \pi } \right\rangle = $ $\frac{{{\varepsilon _0}cE_0^2}}{2}\sqrt {\frac{\sigma }{{2{\varepsilon _0}{\omega _0}}}} \left( {c{h^2}\sqrt {\frac{{2{\varepsilon _0}{\omega _0}}}{\sigma }} - {{\sin }^2}\left( {(2n + 1)\frac{\pi }{2} - \sqrt {\frac{{2{\varepsilon _0}{\omega _0}}}{\sigma }} } \right)} \right){\vec e_z}$.
En remplaçant le carré du sinus par celui du cosinus de l’angle complémentaire et en effectuant un développement limité au premier ordre en $\frac{{{\varepsilon _0}{\omega _0}}}{{2\sigma }}$, on obtient: $\left\langle {\vec \pi } \right\rangle = \frac{{{\varepsilon _0}cE_0^2}}{4}\sqrt {\frac{{2{\varepsilon _0}{\omega _0}}}{\sigma }} {\vec e_z}$.
La densité moyenne d’énergie est $\left\langle w \right\rangle = \frac{{{\varepsilon _0}}}{4}\vec E.{\vec E^*} + \frac{{{\varepsilon _0}{c^2}}}{4}\vec B.{\vec B^*}$ et après calcul $\left\langle w \right\rangle = \frac{{{\varepsilon _0}E_0^2}}{4}ch2\sqrt {\frac{{2{\varepsilon _0}{\omega _0}}}{\sigma }} \frac{z}{Z}$; on peut alors calculer l’énergie dans la cavité par intégration sur son volume:
$W = \int\limits_{ - \frac{Z}{2}}^{\frac{Z}{2}} {wSdz} $$ = \frac{{{\varepsilon _0}E_0^2S}}{4}\frac{{sh\sqrt {\frac{{2{\varepsilon _0}{\omega _0}}}{\sigma }} }}{{\frac{1}{Z}\sqrt {\frac{{2{\varepsilon _0}{\omega _0}}}{\sigma }} }}$ et par développement limité au premier ordre: $W = \frac{{{\varepsilon _0}E_0^2}}{4}SZ$
La puissance perdue dans le métal est $P = 2\left\langle {\vec \pi .\vec S} \right\rangle $ $ = 2Wc\sqrt {\frac{{2{\varepsilon _0}{\omega _0}}}{\sigma }} $ en tenant compte des deux faces dans la cavité et par ailleurs $P = - \frac{{dW}}{{dt}}$; on obtient une équation différentielle linéaire du premier ordre à coefficients constants en W dont l’intégration donne $W = {W_0}{e^{ - \frac{t}{{\tau '}}}}$ avec $\tau ' = \sqrt {\frac{\sigma }{{2{\varepsilon _0}{\omega _0}}}} \frac{Z}{{2c}} = \frac{\tau }{2}$ ($\tau $ de 5.15°); quand on passe de l’amplitude à l’énergie, l’exposant est divisé par 2; l’énergie varie plus rapidement que l’amplitude.
6.1°) L’onde est voisine d’une onde plane: la dépendance en z (passage par toutes les valeurs de ${e^{ - ikz}}$) est contenue dans ${e^{ - ikz}}$ dont la variation se fait sur la longueur caractéristique $\lambda $(longueur d’onde); u varie donc peu sur une distance de quelques longueurs d’onde.
Par ailleurs, le faisceau est étroit et donc limité latéralement; u varie rapidement avec x et y alors qu’il varie lentement en z. D’où: $\left| {\frac{{{\partial ^2}u}}{{\partial {z^2}}}} \right| < < \left| {\frac{{{\partial ^2}u}}{{\partial {x^2}}}} \right|\;et\;\left| {\frac{{{\partial ^2}u}}{{\partial {y^2}}}} \right|$ et $\left| {\frac{{\partial u}}{{\partial z}}} \right| < < \frac{{\left| u \right|}}{\lambda }$
6.2°) Le champ électrique vérifie: $\frac{{{\partial ^2}E}}{{\partial {x^2}}} + \frac{{{\partial ^2}E}}{{\partial {y^2}}} + \frac{{{\partial ^2}E}}{{\partial {z^2}}} + \frac{{{\omega ^2}}}{{{c^2}}}E = 0$; il suffit de le remplacer par $u(x,y,z){e^{i(\omega t - kz)}}$ avec $k = \frac{\omega }{c}$ pour obtenir $\frac{{{\partial ^2}u}}{{\partial {x^2}}} + \frac{{{\partial ^2}u}}{{\partial {y^2}}} + \frac{{{\partial ^2}u}}{{\partial {z^2}}} - 2ik\frac{{\partial u}}{{\partial z}} = 0$; or $k = \frac{{2\pi }}{\lambda }$; en tenant compte des approximations: $\frac{{{\partial ^2}u}}{{\partial {x^2}}} + \frac{{{\partial ^2}u}}{{\partial {y^2}}} - 2ik\frac{{\partial u}}{{\partial z}} = 0$ équation d’onde paraxiale.
6.3°) Pour une onde sphérique divergente en M(x,y,z), issue de O(0,0,0), en appelant r la distance à OM le champ est $E = \frac{{{E_0}}}{r}{e^{ - ikr}}{e^{i\omega t}}$ avec $r = \sqrt {{x^2} + {y^2} + {z^2}} $; en z>0 $E = \frac{{{E_0}}}{{\sqrt {{x^2} + {y^2} + {z^2}} }}{e^{ikz\left( {1 - \sqrt {1 + \frac{{{x^2} + {y^2}}}{{{z^2}}}} } \right)}}{e^{i(\omega t - kz)}}$ d’où ${u_S}(x,y,z) = \frac{{{E_0}}}{{\sqrt {{x^2} + {y^2} + {z^2}} }}{e^{ikz\left( {1 - \sqrt {1 + \frac{{{x^2} + {y^2}}}{{{z^2}}}} } \right)}}$. Pour une onde sphérique convergente en O, en z<0, on aurait la même expression.
6.4°) Si z >> x et y, alors $u{'_S}(x,y,z) = \frac{{{E_0}}}{z}{e^{ - ik\left( {\frac{{{x^2} + {y^2}}}{{2z}}} \right)}}$
6.5°) $\frac{{{\partial ^2}u{'_S}}}{{\partial {x^2}}} = - ik\frac{{{E_0}}}{{{z^2}}}{e^{ - ik\left( {\frac{{{x^2} + {y^2}}}{{2z}}} \right)}}\left( {1 - ik\frac{{{x^2}}}{z}} \right)$ et de même en y
d’où $\frac{{{\partial ^2}u{'_S}}}{{\partial {x^2}}} + \frac{{{\partial ^2}u{'_S}}}{{\partial {y^2}}} = - ik\frac{{{E_0}}}{{{z^2}}}{e^{ - ik\left( {\frac{{{x^2} + {y^2}}}{{2z}}} \right)}}\left( {2 - ik\frac{{{x^2} + {y^2}}}{z}} \right)$
Par ailleurs $2ik\frac{{\partial u{'_S}}}{{\partial z}} = ik\frac{{{E_0}}}{{{z^2}}}{e^{ - ik\left( {\frac{{{x^2} + {y^2}}}{{2z}}} \right)}}\left( {2 - ik\frac{{{x^2} + {y^2}}}{z}} \right)$; on en déduit: $\frac{{{\partial ^2}u{'_S}}}{{\partial {x^2}}} + \frac{{{\partial ^2}u{'_S}}}{{\partial {y^2}}} - 2ik\frac{{\partial u{'_S}}}{{\partial z}} = 0$
$u{'_S}$ est solution exacte de l’équation d’onde paraxiale.
6.6°) $u(x,y,z) = A(z){e^{ - ik\left( {\frac{{{x^2} + {y^2}}}{{2q(z)}}} \right)}}$; le même genre de calcul qu’en 6.5° donne: $\frac{{2ik}}{A}\frac{{dA}}{{dz}} + \left( {\frac{{{k^2}}}{{{q^2}}}({x^2} + {y^2})(1 - \frac{{dq}}{{dz}}) + \frac{{2ik}}{q}} \right) = 0$; on cherche A=A(z) et de ce fait le coefficient de ${x^2} + {y^2}$ doit être nul $ \Rightarrow $ $\frac{{dq}}{{dz}} = 1$ et $q(z) = {q_0} + z - {z_0}$; puis $\frac{{dA}}{{dz}} = - \frac{A}{{q(z)}}$ soit en intégrant $A(z)q(z) = cste$ ou $A(z) = {A_0}\frac{{{q_0}}}{{q(z)}}$
6.7°) $u(x,y,z) = \frac{{{A_0}{q_0}}}{{{q_0} + z - {z_0}}}{e^{ - ik\left( {\frac{{{x^2} + {y^2}}}{{2(q + z - {z_0})}}} \right)}}$ et selon le texte $u(x,y,z) = {u_0}B{e^{ - ikB\left( {\frac{{{x^2} + {y^2}}}{2}} \right)}}$ avec $B = \frac{1}{{R(z)}} - \frac{{2i}}{{k{w^2}(z)}}$; en identifiant les phases on en déduit $B = \frac{1}{{R(z)}} - \frac{{2i}}{{k{w^2}(z)}} = \frac{1}{{q(z)}} = \frac{1}{{{q_0} + z - {z_0}}}$ et en identifiant les amplitudes ${u_0} = {A_0}{q_0}$.
6.8°) On a ${q_0} = {q_1} + i{q_2}$ avec ${q_1} + z - {z_0} = \frac{{{{(k{w^2})}^2}R}}{{{{(k{w^2})}^2} + 4{R^2}}}$ et ${q_2} = \frac{{2k{w^2}{R^2}}}{{{{(k{w^2})}^2} + 4{R^2}}}$
d’où $R(z) = \frac{{q_2^2}}{{{q_1} + z - {z_0}}} + {q_1} + z - {z_0}$ et ${w^2}(z) = \frac{{2{q_2}}}{k}\left( {1 + \frac{{{{({q_1} + z - {z_0})}^2}}}{{q_2^2}}} \right)$
R(z) représente le rayon de courbure de l’onde; il varie avec z; l’onde est plane si R devient infini et donc si ${q_1} = 0$ et $z = {z_0}$; on pose ${w_0} = w({z_0})$ et on obtient ${q_2} = \frac{{kw_0^2}}{2} = {Z_0}$ (longueur de Rayleigh); avec ces notations: ${Z_0} = {q_2} = \frac{{\pi w_0^2}}{\lambda }$ et ${q_0} = i{Z_0}$ puis $R(z) = \frac{{Z_0^2}}{{z - {z_0}}} + z - {z_0}$ et $w(z) = {w_0}{\left( {1 + {{\left( {\frac{{z - {z_0}}}{{{Z_0}}}} \right)}^2}} \right)^{\frac{1}{2}}}$
6.9°) En remplaçant dans l’expression du champ électrique, on trouve: $E = {A_0}\frac{{{e^{ - iArc\tan \left( {\frac{{z - {z_0}}}{{{Z_0}}}} \right)}}}}{{{{\left[ {1 + {{\left( {\frac{{z - {z_0}}}{{{Z_0}}}} \right)}^2}} \right]}^{\frac{1}{2}}}}}{e^{ - \frac{{ik{r^2}}}{{2\left( {z - {z_0} + \frac{{Z_0^2}}{{z - {z_0}}}} \right)}}}}{e^{ - \frac{{k{r^2}}}{{2\left[ {1 + {{\left( {\frac{{z - {z_0}}}{{{Z_0}}}} \right)}^2}} \right]{Z_0}}}}}{e^{i(\omega t - kz)}}$ avec $r = \sqrt {{x^2} + {y^2}} $
Pour une surface d’onde, la phase est constante d’où: $kz + Arc\tan \left( {\frac{{z - {z_0}}}{{{Z_0}}}} \right) + \frac{{k{r^2}}}{{2\left( {z - {z_0} + \frac{{Z_0^2}}{{z - {z_0}}}} \right)}} = cste$
w(z) est la distance, selon une direction perpendiculaire à l’axe du faisceau, au bout de laquelle l’amplitude est divisée par e et l’intensité par e2; on l’appelle le rayon focal: il est fonction de z.
6.10°) $w\frac{{dw}}{{dz}} = w_0^2\frac{{z - {z_0}}}{{{Z_0}}}$ s’annule si $z = {z_0}$; on a un minimum de w(z); en cet endroit le faisceau a une « section » minimale de rayon focal ${w_0}$(« waist » en anglais) et le rayon de courbure est infini. Le texte parle de « foyer », mais le nom est incorrect.
L’approximation $\left| {\frac{{\partial u}}{{\partial z}}} \right| < < \frac{{\left| u \right|}}{\lambda }$ est vérifiée si $r < < \frac{{kw_0^2}}{2}$.
Enfin d’après l’expression du champ de 6.9°, l’abscisse z0 de la zone à rayon focal minimal et la valeur w0 de ce minimum déterminent totalement la géométrie du faisceau.
6.11°) L’onde en sortie est sphérique convergente de centre le foyer image F.
6.12°) On note e l’épaisseur de la lentille mince et n l’indice du verre dont elle est faite. Dans le cadre de l’approximation de Gauss, un rayon incident parallèle à l’axe traverse la lentille mince selon le segment IJ très court (inférieur à e) par rapport aux rayons de courbure R1 et R2 des faces et peu incliné par rapport à l’axe de la lentille. Le retard de phase introduit par la présence de la lentille est donc $\phi (r) = k(n - 1)IJ$. Soit en appelant H et H’ les projections de I et J sur l’axe et S1 et S2 les sommets des faces: $\phi (r) = k(n - 1)(e - \overline {{S_1}H} - \overline {H'{S_2}} )$; pour évaluer les deux segments, on raisonne sur le cercle ci-contre.
Soit α l’angle entre SS’ et IS’ qu’on retrouve entre IS et IH: $\tan \alpha = \frac{r}{{2R - \overline {SH} }} = \frac{{\overline {SH} }}{r}$ $ \Rightarrow $ $\overline {SH} \approx \frac{{{r^2}}}{{2R}}$ $ \Rightarrow $
$\phi (r) = k(n - 1)\left( {e - \frac{{{r^2}}}{2}\left( {\frac{1}{{{R_1}}} + \frac{1}{{{R_2}}}} \right)} \right)$ ; or la vergence de la lentille est $V = \frac{1}{{f'}} = (n - 1)\left( {\frac{1}{{{R_1}}} + \frac{1}{{{R_2}}}} \right)$ d’où $\phi (r) = \phi (0) - k\frac{{{V^2}}}{2}{r^2}$
Les rayons de courbure sont comptés avec la convention: ${R_1} = \overline {{S_1}{C_1}} $ et ${R_2} = - \overline {{S_2}{C_2}} $(attention au signe moins)
6.13°) Soit un point A sur z’z émettant une onde sphérique divergente avec $\overline {OA} = {z_1}$.
$E = \frac{{{E_0}}}{{\left| {z - {z_1}} \right|}}{e^{ - ik\frac{{{r^2}}}{{2(z - {z_1})}}}}{e^{i(\omega t - kz)}}$ pour $z > {z_1}$; en z=0 et après traversée de la lentille, $E = \frac{{{E_0}}}{{\left| {{z_1}} \right|}}{e^{ - ik\frac{{{r^2}}}{{2{z_1}}}}}{e^{ - i(\phi (r) - \phi (0))}}{e^{i(\omega t - \phi (0))}}$$ = \frac{{{E_0}}}{{\left| {{z_1}} \right|}}{e^{ik\frac{{{r^2}}}{{2{z_1}}}\left( {\frac{1}{{{z_1}}} + \frac{1}{{f'}}} \right)}}{e^{i(\omega t - \phi (0))}}$expression compatible avec une onde sphérique divergente centrée en A’, d’abscisse z2 positive qui s’écrit: $E = \frac{{\underline {E{'_0}} }}{{\left| {{z_2}} \right|}}{e^{ik\frac{{{r^2}}}{{2{z_2}}}}}{e^{i\omega t}}$; on ne cherche pas la relation entre les amplitudes complexes; il suffit d’identifier les phases et ce pour toute valeur de r dans le cadre de l’approximation de Gauss. On en déduit: $\frac{1}{{{z_2}}} - \frac{1}{{{z_1}}} = \frac{1}{{f'}} = V$; on peut généraliser à un système centré mince quelconque (indices extrêmes n et n’); il introduit un retard de phase $\phi (r) = \phi (0) - {k_0}\frac{{{V^2}}}{2}{r^2}$ ce qui conduit à $\frac{{f'}}{{{z_2}}} + \frac{f}{{{z_1}}} = 1$ avec $V = \frac{{n'}}{{f'}} = - \frac{n}{f}$; pour un dioptre sphérique: $V = \frac{{n' - n}}{{\overline {SC} }}$ et pour un miroir sphérique $V = \frac{{ - 2n}}{{\overline {SC} }}$.
6.14°) Pour l’onde incidente en z=0, ${E_1}(x,y,0,t) = {u_{01}}\left( {\frac{1}{{{R_1}(0)}} - \frac{{2i}}{{kw_1^2(0)}}} \right){e^{ - \frac{{ik{r^2}}}{{2{R_1}(0)}}}}{e^{ - \frac{{{r^2}}}{{w_1^2(0)}}}}{e^{i\omega t}}$
Juste après traversée de la lentille: ${E_2}(x,y,0,t) = {u_{01}}\left( {\frac{1}{{{R_1}(0)}} - \frac{{2i}}{{kw_1^2(0)}}} \right){e^{ - \frac{{ik{r^2}}}{{2{R_1}(0)}}}}{e^{\frac{{ikV{r^2}}}{2}}}{e^{ - \frac{{{r^2}}}{{w_1^2(0)}}}}{e^{i\omega t}}$ ce qui est compatible avec une onde émergente du même type: ${E_2}(x,y,0,t) = {u_{02}}\left( {\frac{1}{{{R_2}(0)}} - \frac{{2i}}{{kw_2^2(0)}}} \right){e^{ - \frac{{ik{r^2}}}{{2{R_2}(0)}}}}{e^{ - \frac{{{r^2}}}{{w_1^2(0)}}}}{e^{i\omega t}}$
En identifiant les exposants imaginaires en r2, on a: $\frac{1}{{{R_1}(0)}} - \frac{1}{{{R_2}(0)}} = V$ et ${w_1}(0) = {w_2}(0)$; la première relation est la relation de conjugaison des « foyers » et la seconde exprime la continuité du rayon focal sur la lentille.
On a $\frac{1}{{{q_1}(0)}} - \frac{1}{{{q_2}(0)}} = V$; ${R_2}(0) = - \left( {\frac{{Z_2^2}}{{{z_2}}} + {z_2}} \right)$ et ${R_1}(0) = - \left( {\frac{{Z_1^2}}{{{z_1}}} + {z_1}} \right)$; ${Z_1} = \frac{{kw_1^2}}{2}$ et ${Z_2} = \frac{{kw_2^2}}{2}$;
${w_1}(0) = {w_1}{\left( {1 + {{\left( {\frac{{{z_1}}}{{{Z_1}}}} \right)}^2}} \right)^{\frac{1}{2}}}$et ${w_2}(0) = {w_2}{\left( {1 + {{\left( {\frac{{{z_2}}}{{{Z_2}}}} \right)}^2}} \right)^{\frac{1}{2}}}$
On obtient: $w_2^2\left( {1 + {{\left( {\frac{{{z_2}}}{{{Z_2}}}} \right)}^2}} \right) = w_1^2\left( {1 + {{\left( {\frac{{{z_1}}}{{{Z_1}}}} \right)}^2}} \right)$ et $\frac{1}{{{z_2} + \frac{{Z_2^2}}{{{z_2}}}}} - \frac{1}{{{z_1} + \frac{{Z_1^2}}{{{z_1}}}}} = V = \frac{1}{{f'}}$
Comme en 6.13° pour un système mince $\frac{{f'}}{{{q_2}(0)}} + \frac{f}{{{q_1}(0)}} = 1$.
A partir des 2 équations à 2 inconnues obtenues on peut calculer w2 et z2:
$w_2^2 = \frac{{w_1^2\left( {1 + {{\left( {\frac{{{z_1}}}{{{Z_1}}}} \right)}^2}} \right)}}{{1 + {{\left[ {V\left( {{Z_1} + \frac{{z_1^2}}{{{Z_1}}}} \right) + \frac{{{z_1}}}{{{Z_1}}}} \right]}^2}}}$ et ${z_2} = \frac{{\left[ {1 + {{\left( {\frac{{{z_1}}}{{{Z_1}}}} \right)}^2}} \right]\left( {{z_1} + V(Z_1^2 + z_1^2)} \right)}}{{1 + {{\left[ {V\left( {{Z_1} + \frac{{z_1^2}}{{{Z_1}}}} \right) + \frac{{{z_1}}}{{{Z_1}}}} \right]}^2}}}$
Mais ces expressions ne servent à rien ensuite; merci d’en avoir demandé le calcul!
6.15°) Pour un miroir sphérique: $\frac{1}{{{R_1}(0)}} + \frac{1}{{{R_2}(0)}} = - V = \frac{2}{{\overline {SC} }}$ et ${w_1}(0) = {w_2}(0)$
et $w_2^2\left( {1 + {{\left( {\frac{{{z_2}}}{{{Z_2}}}} \right)}^2}} \right) = w_1^2\left( {1 + {{\left( {\frac{{{z_1}}}{{{Z_1}}}} \right)}^2}} \right)$ et $\frac{1}{{{z_2} + \frac{{Z_2^2}}{{{z_2}}}}} + \frac{1}{{{z_1} + \frac{{Z_1^2}}{{{z_1}}}}} = V = \frac{1}{{f'}}$
Le faisceau est stable si la zone de resserrement est unique (le faisceau résultant a une forme gaussienne) caractérisé par w1 et $Z = \frac{{kw_1^2}}{2}$ et par les distances algébriques z1, z2 aux miroirs avec $L = {z_2} - {z_1}$; à partir de la relation de conjugaison, on obtient: ${z_1} + \frac{{{Z^2}}}{{{z_1}}} = - {R_1}$ et ${z_2} + \frac{{{Z^2}}}{{{z_2}}} = {R_2}$; on remarque que les rayons de courbure du faisceau gaussien au niveau des miroirs sont les rayons de courbure des miroirs.
On a 3 équations à trois inconnues qui permettent de déterminer z1, z2 et w1. On calcule seulement z1, z2.
${z_2} = \frac{{L({R_1} - L)}}{{{R_1} + {R_2} - 2L}}$ ${z_1} = \frac{{L(L - {R_2})}}{{{R_1} + {R_2} - 2L}}$ ${Z^2} = \frac{{L({R_1} - L)({R_2} - L)({R_1} + {R_2} - L)}}{{{{({R_1} + {R_2} - 2L)}^2}}}$
Avec ${g_i} = 1 - \frac{L}{{{R_i}}}$ on peut écrire ${Z^2} = {L^2}\frac{{{g_1}{g_2}(1 - {g_1}{g_2})}}{{{{({g_1} + {g_2} - 2{g_1}{g_2})}^2}}}$>0 $ \Rightarrow $ $0 < {g_1}{g_2} < 1$; c’est la condition de stabilité.
Notons que ${z_1} = - L\frac{{{g_2}(1 - {g_1})}}{{{g_1} + {g_2} - 2{g_1}{g_2}}}$ et ${z_2} = L\frac{{{g_1}(1 - {g_2})}}{{{g_1} + {g_2} - 2{g_1}{g_2}}}$
6.16°) Si les deux rayons de courbure sont égaux: ${Z^2} = \frac{L}{4}(2R - L) = \left( {\frac{{kw_1^2}}{2}} \right)$ $ \Rightarrow $ $w_1^2 = \frac{1}{k}\sqrt {(2R - L)L} $
La condition de stabilité est réalisée si R > L/2.
6.17°) Condition de résonance: sur un aller et retour, le long de z’z en particulier, le déphasage doit être un multiple entier de fois $2\pi $ soit sur un aller seulement un multiple entier de fois $\pi $; or $\varphi (z) = kz - Arc\tan \frac{{{z_0}}}{{{Z_0}}}$ et donc ${\varphi _2} - {\varphi _1} = k({z_2} - {z_1}) - (Arc\tan \frac{{{z_2}}}{Z} - Arc\tan \frac{{{z_1}}}{Z}) = q\pi $, q entier positif ou nul; on pose $\gamma = Arc\tan \frac{{{z_2}}}{Z} - Arc\tan \frac{{{z_1}}}{Z}$; le calcul donne ${\cos ^2}\gamma = {g_1}{g_2}$ puis $\nu = \frac{c}{{2L}}\left( {q + \frac{1}{\pi }Arc\cos \sqrt {{g_1}{g_2}} } \right)$; si les deux rayons sont égaux $\nu = \frac{c}{{2L}}\left( {q + \frac{1}{\pi }Arc\cos \left( {1 - \frac{L}{R}} \right)} \right)$.
7.1°) L’équation de propagation de la composante x du champ électrique est $\frac{{{\partial ^2}{E_x}}}{{\partial {x^2}}} + \frac{{{\partial ^2}{E_x}}}{{\partial {y^2}}} + \frac{{{\partial ^2}{E_x}}}{{\partial {z^2}}} + \frac{{{\omega ^2}}}{{{c^2}}}{E_x} = 0$
${{E}_{x}}=f(x)g(y)h(z)\Rightarrow \frac{f''}{f}+\frac{g''}{g}+\frac{h''}{h}+\frac{{{\omega }^{2}}}{{{c}^{2}}}=0$; pour pouvoir assurer les conditions limites, il faut: $\frac{f''}{f}=-k_{x}^{2}$ $\frac{g''}{g}=-k_{y}^{2}$ et $\frac{h''}{h}=-k_{z}^{2}$ avec $k_x^2 + k_y^2 + k_z^2 = \frac{{{\omega ^2}}}{{{c^2}}}$; avec un des sommets de la cavité à l’origine des axes:
$f(x) = A\cos {k_x}x + B\sin {k_x}x$, $g(y) = C\cos {k_y}y + D\sin {k_y}y$, $h(z) = E\cos {k_z}z + F\sin {k_z}z$
Les conditions limites sont:
${E_x}(x = 0\;et\;x = X) = {E_y}(x = 0\;et\;x = X) = 0$
${E_z}(y = 0\;et\;y = Y) = {E_x}(y = 0\;et\;y = Y) = 0$
${E_x}(z = 0\;et\;z = Z) = {E_y}(z = 0\;et\;z = Z) = 0$
g(0) = g(Y) = 0 et h(0) = h(Z) impliquent C = E = 0
$g(y) = D\sin {k_y}y$ $h(z) = F\sin {k_z}z$
${k_y} = \pm \frac{{m\pi }}{Y}$ ${k_z} = \pm \frac{{n\pi }}{Z}$
${E_x} = ({A_1}\cos {k_x}x + {B_1}\sin {k_x}x)\sin {k_y}y\sin {k_z}z{e^{i\omega t}}$ et de même ${E_y} = \sin k{'_x}x({A_2}\cos k{'_y}y + {B_2}\sin k{'_y}y)\sin k{'_z}z{e^{i\omega t}}$
${{E}_{z}}=\sin k{{''}_{x}}x\sin k{{''}_{y}}y({{A}_{3}}\cos k{{''}_{z}}z+{{B}_{3}}\sin k{{''}_{z}}z){{e}^{i\omega t}}$
Du fait que $div\vec E = 0$, il existe une combinaison linéaire entre les dérivées de ces composantes vraie pour tout x,y,z dans le domaine de la cavité; on peut en conclure qu’on a les mêmes composantes de vecteur d’onde entre les trois composantes du champ et donc que ${k_x} = \pm \frac{{l\pi }}{X}$ ${k_y} = \pm \frac{{m\pi }}{Y}$ ${k_z} = \pm \frac{{n\pi }}{Z}$ avec l,m,n $ \in \aleph $.
La combinaison linéaire est également vraie en x = 0 ou X, en y = 0 ou Y, en z = 0 ou Z; on en déduit ${B_1} = {B_2} = {B_3} = 0$ et ${A_1}l + {A_2}m + {A_3}n = 0$; en fait, il n’existe que deux constantes indépendantes;
${E_x} = {A_1}\cos {k_x}x\sin {k_y}y\sin {k_z}z{e^{i\omega t}}$ ${E_y} = {A_2}\sin {k_x}x\cos {k_y}y\sin {k_z}z{e^{i\omega t}}$ ${E_z} = {A_3}\sin {k_x}x\sin {k_y}y\cos {k_z}z{e^{i\omega t}}$
On peut également calculer les composantes du champ magnétique et vérifier que le champ magnétique a des composantes normales nulles sur les faces de la cavité.
Enfin $k_x^2 + k_y^2 + k_z^2 = \frac{{{\omega ^2}}}{{{c^2}}}$ implique $\nu = \frac{c}{2}{\left( {\frac{{{l^2}}}{{{X^2}}} + \frac{{{m^2}}}{{{Y^2}}} + \frac{{{n^2}}}{{{Z^2}}}} \right)^{\frac{1}{2}}}$.
7.2°) Dans l’espace des vecteurs d’onde, les extrémités des vecteurs d’onde possibles définissent un réseau cubique dont la maille élémentaire à les cotés $\frac{\pi }{X},\frac{\pi }{Y},\frac{\pi }{Z}$ de volume $v = \frac{{{\pi ^3}}}{{XYZ}} = \frac{{{\pi ^3}}}{V}$; la zone sphérique de rayons k et k+dk a le volume $d\tau = 4\pi {k^2}dk$; un mode correspond à une onde progressive qui se réfléchit sur les différentes faces soit à $ \pm l, \pm m, \pm n$; il faut compter un huitième du volume de la zone sphérique; en plus, pour l,m,n donnés on a deux états de polarisation indépendants (${A_1} = 0$ ou ${A_2} = 0$); le nombre de modes de norme de vecteur d’onde compris entre k et k+dk est $dn = \frac{2}{8}\frac{{4\pi {k^2}dk}}{{{\pi ^3}}}V$; c’est aussi le nombre de modes de fréquence comprise entre $\nu \;et\;\nu + d\nu $$ \Rightarrow $ $dn = 8\pi V\frac{{{\nu ^2}}}{{{c^3}}}d\nu $ et ${\rm N}(\nu ) = V\frac{{8\pi {\nu ^2}}}{{{c^3}}}$
7.3°) Le calcul de I fait intervenir des intégrales du type $\int\limits_0^X {\cos {n_1}\frac{\pi }{X}\cos {n_2}\frac{\pi }{X}dx} $ ou bien en sinus, qu’on linéarise en intégrales de cosinus faisant figurer la somme ou la différence des deux entiers; ces intégrales sont nulles si les entiers sont différents. De ce fait $I = 0$.
7.4°) Les champs électrique et magnétique sont combinaison linéaire des champs des modes propres; quand on les remplace dans l’expression de la densité d’énergie électromagnétique on fait intervenir en développant des intégrales de produits de champs qui sont nulles et des intégrales proportionnelles au carré des champs des modes propres; pour chaque mode un terme est proportionnel au carré du champ électrique et un autre au carré du champ magnétique; on a deux termes quadratiques par mode propre; en retenant, selon le théorème d’équipartition de l’énergie, l’énergie kT/2 par degré de liberté, on a l’énergie kT par mode propre.
On retrouve le théorème de Rayleigh-Jeans qui dit que chaque mode est équivalent à un oscillateur harmonique linéaire. D’où:
$dU(\nu ) = {\rm N}(\nu )d\nu = V\frac{{8\pi {\nu ^2}kT}}{{{c^3}}}d\nu $ soit une densité d’énergie spectrale $u(\nu ) = \frac{{8\pi {\nu ^2}kT}}{{{c^3}}}$.
7.5°) ${U_t} = \int\limits_0^\infty {U(\nu )d\nu } = \frac{{\pi kTV}}{{{c^3}}}\int\limits_0^\infty {{\nu ^2}d\nu } $ ; l’intégrale n’est pas définie; l’énergie emmagasinée ne peut être infinie.
7.6°) La probabilité de l’énergie ${E_n} = nh\nu $ pour le mode propre de fréquence $\nu $ est ${P_n} = A{e^{ - \frac{{nh\nu }}{{kT}}}}$; or $\sum\limits_{n = o}^\infty {{P_n}} = 1$(ne pas oublier n=0; la probabilité de l’état d’énergie nulle est non nulle); d’où ${P_n} = \frac{{{e^{ - \frac{{nh\nu }}{{kT}}}}}}{{\sum\limits_{n = 0}^\infty {{e^{ - \frac{{nh\nu }}{{kT}}}}} }}$
7.7°) L’énergie moyenne du mode de fréquence $\nu $ est ${\rm E}(\nu ) = \sum\limits_{n = o}^\infty {{P_n}} {E_n}$ $ = \frac{{\sum\limits_{n = 0}^\infty {nh\nu {e^{ - \frac{{nh\nu }}{{kT}}}}} }}{{\sum\limits_{n = 0}^\infty {{e^{ - \frac{{nh\nu }}{{kT}}}}} }}$
$x = \frac{{h\nu }}{{kT}}$ $ \Rightarrow $${\rm E}(\nu ) = xkT\frac{{\sum\limits_{n = 0}^\infty {n{e^{ - nx}}} }}{{\sum\limits_{n = 0}^\infty {{e^{ - nx}}} }}$$ = - xkT\frac{{\frac{d}{{dx}}\sum\limits_{n = 0}^\infty {n{e^{ - nx}}} }}{{\sum\limits_{n = 0}^\infty {{e^{ - nx}}} }}$$ = - xkT\frac{d}{{dx}}\ln \sum\limits_{n = 0}^\infty {{e^{ - nx}}} $$ = - xkT\frac{d}{{dx}}\ln \frac{1}{{1 - {e^{ - x}}}}$$ = kT\frac{x}{{{e^x} - 1}}$
${\rm E}(\nu ) = \frac{{h\nu }}{{{e^{\frac{{h\nu }}{{kT}}}} - 1}}$.
En fait cette démonstration n’est pas valable car les photons obéissent à la statistique de Bose-Einstein et l’énergie d’un oscillateur linéaire n’est pas ${E_n} = nh\nu $ mais ${E_n} = (n + \frac{1}{2})h\nu $; dans l’état fondamental, à une température absolue de O K, avec la formule du texte, l’oscillateur à une énergie nulle; c’est-à-dire que la somme de l’énergie cinétique $\frac{1}{2}m{v^2}$ et de l’énergie potentielle $\frac{1}{2}k{x^2}$est nulle; de ce fait vitesse et abscisse sont nulles simultanément ce qui est contraire au principe d’indétermination de Heisenberg.
7.8°) $dU = \frac{{8\pi V{\nu ^2}}}{{{c^3}}}\frac{{h\nu }}{{{e^{\frac{{h\nu }}{{kT}}}} - 1}}d\nu $$ \Rightarrow $ ${u_\nu } = V\frac{{8\pi h{\nu ^3}/{c^3}}}{{{e^{\frac{{h\nu }}{{kT}}}} - 1}}$; c’est la loi de Planck, relative au corps noir (intérieur d’un four isotherme); l’énergie du corps noir est ${U_t} = V\frac{{2\pi h}}{{{c^3}}}\int\limits_{\nu = 0}^\infty {\frac{{{\nu ^3}}}{{{e^{\frac{{h\nu }}{{kT}}}} - 1}}} \;d\nu $; elle est finie car la fonction se comporte comme ${\nu ^3}{e^{ - \frac{{h\nu }}{{kT}}}}$ quand la fréquence tend vers l’infini et que l’exponentielle l’emporte.
7.9°) On peut définir la densité d’énergie totale ${u_t} = \frac{1}{V}\int\limits_0^\infty {U(\nu )d\nu } $, d’une part car l’intégrale qui l’exprime est définie mathématiquement et d’autre part car on a équilibre thermique; l’énergie se répartit sur tous les modes et se répartit également dans l’espace de la cavité.
7.10°) $x = \frac{{h\nu }}{{kT}}$ ${u_t} = \frac{{8\pi h}}{{{c^3}}}{\left( {\frac{{kT}}{h}} \right)^4}\int\limits_0^\infty {\frac{{{x^3}dx}}{{{e^x} - 1}}} $ soit ${u_t} = \sigma {T^4}$
avec $\sigma = \frac{{8{\pi ^5}{k^4}}}{{15{h^3}{c^3}}}$ ; c’est la loi de Stéphan-Boltzmann; la densité d’énergie totale est proportionnelle à la puissance quatrième de la température absolue.

Concours Physique ENS Ulm, Lyon, Cachan (bio) 1995 (Corrigé)


MECANIQUE DES FLUIDES VISQUEUX: Loi de Poiseuille
(ENS Ulm, Lyon, Cachan 1995, groupe E/S (= option Bio), Durée 4h)
A) Etablissement de la loi de Poiseuille
1) a) Soit le système des coordonnées cylindriques, repérées à partir de la base orthonormée directe $\left( {O,\;{{\vec u}_r},\;{{\vec u}_\theta },\;{{\vec u}_y}} \right)$, telle que $\left( {{{\vec u}_z},\;{{\vec u}_r}} \right)\; = \;\theta $.
Posons: PA = PB = P0, la pression sur l’axe du cylindre.
µ, la masse volumique du fluide.
Calculons:

* La force pressante s’exerçant sur la base passant par A:
${\vec F_A}\; = \;{\vec u_y}\;\int_{r = 0}^r {\int_{\theta = 0}^{2\pi } {\;\left( {{P_0}\; - \;\mu \;g\;r\;\cos \theta } \right)\;r\;dr\;d\theta } } \; = \;{P_0}\;\pi {r^2}\;{\vec u_y}$
* La force pressante s’exerçant sur la base passant par B:
${\vec F_B}\; = \; - \;{\vec u_y}\;\int_{r = 0}^r {\int_{\theta = 0}^{2\pi } {\;\left( {{P_0}\; - \;\mu \;g\;r\;\cos \theta } \right)\;r\;dr\;d\theta } } \; = \; - \;{P_0}\;\pi {r^2}\;{\vec u_y}$
* La force pressante s’exerçant sur la surface latérale:
$\begin{array}{l}{{\vec F}_{lat}}\; = \;\int_{y = 0}^l {\int_{\theta = 0}^{2\pi } {\;\left( {{P_0}\; - \;\mu \;g\;r\;\cos \theta } \right)\;dy\;r\;d\theta \;{{\vec u}_r}} } \;avec\;{{\vec u}_r}\; = \;\cos \;\theta \;{{\vec u}_z}\; + \;\sin \;\theta \;{{\vec u}_x}\\{{\vec F}_{lat}}\; = \;\pi {r^2}\;l\;\mu \;g\;{{\vec u}_z}\;\end{array}$
* La force pressante résultante s’exerçant sur le cylindre vaut:
$\;\;{{\rm{\vec F}}_{\rm{P}}}\; = \;\pi {{\rm{r}}^{\rm{2}}}\;{\rm{l}}\;\mu \;{\rm{g}}\;{{\rm{\vec u}}_{\rm{z}}}\;\;$
1) b) On retrouve le théorème d’Archimède: La force pressante est égale à l’opposé du poids du volume de fluide déplacé.
2) a) Les forces qui s’exercent sur le cylindre sont:
* la force pressante: ${{\rm{\vec F}}_{\rm{P}}}\; = \;\pi {{\rm{r}}^{\rm{2}}}\;{\rm{l}}\;\mu \;{\rm{g}}\;{{\rm{\vec u}}_{\rm{z}}}$
* le poids: ${\rm{\vec P}}\; = \; - \;\pi {{\rm{r}}^{\rm{2}}}\;{\rm{l}}\;\mu \;{\rm{g}}\;{{\rm{\vec u}}_{\rm{z}}}$
Leur résultante est nulle, comme pour tout mouvement rectiligne uniforme.
2) b) Pendant la durée dt, le volume de fluide qui traverse une section droite πa2 vaut πa2 v dt. Donc, le débit volumique vaut $\;\;Q\; = \;\pi {a^2}\;v\;\;$
3) a) Les forces qui s’exercent sur le cylindre sont:
* La force pressante s’exerçant sur la base passant par A:
${\vec F_A}\; = \;{\vec u_y}\;\int_{r = 0}^r {\int_{\theta = 0}^{2\pi } {\;\left( {{P_A}\; - \;\mu \;g\;r\;\cos \theta } \right)\;r\;dr\;d\theta } } \; = \;{P_A}\;\pi {r^2}\;{\vec u_y}$
* La force pressante s’exerçant sur la base passant par B:
${\vec F_B}\; = \; - \;{\vec u_y}\;\int_{r = 0}^r {\int_{\theta = 0}^{2\pi } {\;\left( {{P_B}\; - \;\mu \;g\;r\;\cos \theta } \right)\;r\;dr\;d\theta } } \; = \; - \;{P_B}\;\pi {r^2}\;{\vec u_y}$
* La force pressante s’exerçant sur la surface latérale: $\begin{array}{l}{{\vec F}_{lat}}\; = \;\int_{y = 0}^l {\int_{\theta = 0}^{2\pi } {\;\left( {\left( {{P_A}\; - \;\frac{{\left( {{P_A}\; - \;{P_B}} \right)\;y}}{l}} \right)\; - \;\mu \;g\;r\;\cos \theta } \right)\;dy\;r\;d\theta \;{{\vec u}_r}} } \;avec\;{{\vec u}_r}\; = \;\cos \;\theta \;{{\vec u}_z}\; + \;\sin \;\theta \;{{\vec u}_x}\\{{\vec F}_{lat}}\; = \;\pi {r^2}\;l\;\mu \;g\;{{\vec u}_z}\;\end{array}$
* le poids:
${\rm{\vec P}}\; = \; - \;\pi {{\rm{r}}^{\rm{2}}}\;{\rm{l}}\;\mu \;{\rm{g}}\;{{\rm{\vec u}}_{\rm{z}}}$
* la force de frottement visqueux:
$\begin{array}{l}{{\vec F}_f}\; = \;\int_{y = 0}^l {\int_{\theta = 0}^{2\pi } {\; - \;\eta \;\left| {\frac{{dv}}{{dr}}} \right|\;dy\;r\;d\theta \;{{\vec u}_y}} } \;\\{{\vec F}_f}\; = \; + \;2\pi r\;l\;\eta \;\frac{{\partial v}}{{\partial r}}\;{{\vec u}_y}\;\end{array}$
car, par suite du frottement visqueux, v est une fonction décroissante de r.
3) b) Appliquons le principe fondamental de la dynamique au cylindre, en projection sur l’axe Oy:
$\pi {r^2}\;l\;\mu \;\frac{{\partial v}}{{\partial t}}\;{\vec u_y}\; = \;\pi {r^2}\;\left( {{P_A}\; - \;{P_{\bf{B}}}} \right)\;{\vec u_y}\; + \;2\pi r\;l\;\eta \;\frac{{\partial v}}{{\partial r}}\;{\vec u_y}$
En régime permanent, v ne dépend plus que de r. Donc:
$\;\;\frac{{dv}}{{dr}}\; = \; - \;\frac{{{P_A}\; - \;{P_B}}}{{2\;l\;\eta }}\;r\;\;$
3) c) En tenant compte du fait que v = 0 pour r = a, on obtient:
$\;\;v\; = \;\frac{{{P_A}\; - \;{P_B}}}{{4\;l\;\eta }}\;\left( {{a^2}\; - \;{r^2}} \right)\; = \;{v_{\max }}\;\left( {1\; - \;\frac{{{r^2}}}{{{a^2}}}} \right)\;\;$

4) a) $dQ\; = \;2\pi \;r\;dr\;v(r)\; = \;\pi \;\frac{{{P_A}\; - \;{P_B}}}{{2\;l\;\eta }}\;\left( {{a^2}r\; - \;{r^3}} \right)\;dr$
4) b) $\;\;Q\; = \;\int_{r = 0}^a {dQ} \; = \;\frac{{\pi {a^4}\;\left( {{P_A}\; - \;{P_B}} \right)}}{{8\;l\;\eta }}\;\;$
4) c) D’après (A,2,b), vm = Q/πa2.
D’après (A,3,c) et (A,4,b), vmax = 2Q/πa2.
Donc: vmax = 2 vm.

B) Analogie électrique
1) On peut faire correspondre:
* résistance électrique Re et résistance hydraulique R
* intensité électrique I = charge qui traverse une section par unité de temps, et débit volumique Q = volume qui travers une section par unité de temps
* chute de tension VA - VB et perte de charge PA - PB
* force de frottement visqueux s’exerçant sur les électrons en régime permanent, et responsable de l’effet Joule, et force de frottement visqueux s’exerçant sur le fluide, et responsable de l’échauffement du fluide.
Une différence cependant: la vitesse de déplacement des électrons est uniforme sur toute la section droite en régime permanent.
2) * Association série:
Le débit masse Q est commun.
PA - PB = R1 Q + R2 Q + ... + Rn Q = (Σk Rk) Q = R Q.
On retrouve la loi d’association série des résistances: R = Σk Rk.
* Association parallèle:
La perte de charge PA - PB est commune.
PA - PB = R1 Q1 = R2 Q2 = ... = Rn Qn
Q = Σk Qk = (PA - PB) (Σk 1/Rk) = (PA - PB)/R
On retrouve la loi d’association parallèle des resistances: 1/R = (Σk 1/Rk).
3) Pour un conducteur électrique filiforme, Re = ρ l / π a2.
En remplaçant Q par son expression au sein de la loi de Poiseuille, on obtient
R = 8 η l / π a4.
La différence entre les deux expressions, qui se situe au niveau de l’exposant de a, provient du fait que la vitesse d’écoulement d’un fluide visqueux n’est pas uniforme sur une section droite, tandis qu’elle l’est pour les électrons dans un conducteur homogène en régime permanent.

C) Quelques applications à la circulation sanguine
1) P = µHg g h ⇒ Ps = 17,3 kPa
Pd = 10,7 kPa
2) a) lg v + lg S = cte ⇔ v.S = cte ⇔ Q = cte
Aorte Capillaires Veine cave
v (m.s-1) 0,3 4,2.10-4 0,2
S (m2) 3.10-4 0,22 4,4.10-4
Q (cm3.s-1) 90 92 88
2) b) τ = l/v = lS/vS = V/Q
Le volume sanguin total est environ 4,5 l. Mais il y a deux réseaux. Pour celui qui nous intéresse, on déduit τ ≈ 30 s.
3) a)
Aorte Capillaires Veine cave
R = (PA - PB)/Q (Pa.s.m-3) 1,11.108 0,11.108 0,17.108
3) b) R = 8 η l / π a4 = 2,9.1014 Pa.s.m-3.
3) c) En appliquant la loi d’association parallèle des résistances, on obtient: n = R/Rc = 26 millions de capillaires, ce qui correspond à une section de 13 cm2; le schéma indique 4 cm2. L’ordre de grandeur est donc respecté.
4) R = 8 η l / π a4 = 1,7.1010 Pa.s.m-3.
PA - PB = µ’ g h’ = 8 kPa.
Q = (PA - PB)/R = 0,47 cm3.s-1 = 0,5 % du débit sanguin.
D) Régulation dans le système cardio-vasculaire

1) La loi de Poiseuille nous indique que l = α a4 ΔP / η Q.
Le facteur le plus important est le rayon des vaisseaux (4 fois plus important que les autres, en différentielle logarithmique).
2) En appliquant , on obtient R1 = 5,0.108 Pa.s.m-3.
R2 = 3,1.108 Pa.s.m-3.
R3 = 5,0.108 Pa.s.m-3.
En appliquant la loi d’association parallèle des résistances, on obtient R0 = 1,4.108 Pa.s.m-3.
3) En appliquant la loi d’association parallèle des résistances, on obtient
R’ = 1,0.108 Pa.s.m-3.
On en déduit ΔP’ = R’/Q0 = 9,4 kPa, donc Q’1 = ΔP’/R1 = 18,8 cm3.s-1.
Q’2 = ΔP’/R2 = 15,0 cm3.s-1.
Q’3 = ΔP’/R3 = 56,3 cm3.s-1.
4) a) ΔP = (Q0 - Q)/α + ΔP0.
Q’ = Q0 - α (ΔP’ - ΔP0) = 287 cm3.s-1.

4) b) ΔP’’ = R’ Q’’ = R’ [Q0 - α (ΔP’’ - ΔP0)]
⇔ ΔP’’ = R’ [Q0 + α ΔP0]/(1+R’α) = 12,1 kPa
⇒ Q’’ = Q0 - α (ΔP’’ - ΔP0) = 116 cm3.s-1.
Q’’1 = ΔP’’/R1 = 24,2 cm3.s-1.
Q’’2 = ΔP’’/R2 = 19,3 cm3.s-1.
Q’’3 = ΔP’’/R3 = 72,5 cm3.s-1.
4) c) La dernière situation correspond à une meilleure régulation que la précédente, (M’’ est beaucoup plus proche de M0 que M’, comme le montre le dessin de la question (4,a)), donc à un meilleur fonctionnement de l’organisme.
E) Quelques considérations énergétiques

1) Pm = ΔP0 . S . v = ΔP0 . Q = 1,1 W = 1% de P totale.
Cette puissance sert à faire circuler le sang dans les vaisseaux, et est perdue sous forme de chaleur du fait des frottements.
Elle contribue pour 1 % au maintient de la température du corps. L’essentiel de l’énergie nécessaire à ce maintien est apporté par la combustion des nutriments.
2) a) Pm = ΔP02 πa4 / 8lη.
2) b) La portion de fluide est soumise aux forces de frottements visqueux:
$\begin{array}{l}d{{\vec F}_f}\; = \;\left[ {2\pi \;l\;\eta \;r\;\left( {\frac{{dv}}{{dr}}} \right)} \right]\left( {r + dr} \right)\;{{\vec u}_y}\; - \left[ {2\pi \;l\;\eta \;r\;\left( {\frac{{dv}}{{dr}}} \right)} \right]\left( r \right)\;{{\vec u}_y}\; = \;\left[ { - \;2\pi \;l\;\eta \;r\;\frac{{r\;\Delta {P_0}}}{{2\;l\;\eta }}\;{{\vec u}_y}} \right]_r^{r + dr}\\\;\;\;\;\; = \;\left[ { - \;\Delta {P_0}\;\pi {r^2}\;{{\vec u}_y}} \right]_r^{r + dr}\end{array}$
soit: $\;\;d{\vec F_f}\;\; = \; - \;2\pi r\;\Delta {P_0}\;dr\;{\vec u_y}\;$
Ces frottements consomment, sur la section totale du cylindre, une puissance: $W\; = \;\int_{r = 0}^a { - \;{{\vec F}_f}\;.\;v\;{{\vec u}_y}\; = \;} \int_{r = 0}^a {\;{{(\Delta {P_0})}^2}\;\frac{\pi }{{2\;l\;\eta }}\;\left( {{a^2}\; - \;{r^2}} \right)\;r\;dr = } \;\frac{{\;\pi {a^4}}}{{8\;l\;\eta }}\;{\left( {\Delta {P_0}} \right)^2}\;$, soit:
$\;\;W\; = \;\frac{{{{\left( {\Delta {P_0}} \right)}^2}}}{R}\; = \;{P_m}\;\;$
3) On retrouve une forme analogue à la puissance Joule: (ΔV)2/Re.

Concours Physique ENS Lyon-Cachan M' 1995 (Corrigé)

ENS Lyon-Cachan M’ 1995
Partie A : ondes acoustiques de faible amplitude.
A.1.1. $div(\mu \vec v) + \frac{{\partial \mu }}{{\partial t}} = 0$ (1) A.1.2. $\mu \left( {\frac{{\partial \vec v}}{{\partial t}} + (\vec v.gr\vec ad)\vec v} \right) = - gr\vec ad\,p$ (2)
A.2.1. ${\mu _0}div(\vec v) + \frac{{\partial \mu '}}{{\partial t}} = 0$ (1’) et ${\mu _0}\frac{{\partial \vec v}}{{\partial t}} = - gr\vec ad\,p'$ (2’)
A.2.2. En prenant le rotationnel de (2’) on obtient $\frac{\partial }{{\partial t}}\left( {r\vec ot(\vec v)} \right) = \vec 0$ donc $r\vec ot(\vec v)$ est un vecteur permanent. Si on veut que sa valeur moyenne soit nulle, il faut qu’il soit nul. L’écoulement est alors potentiel et $\vec v = gr\vec ad\phi $. Notons que φ est défini à une fonction du temps (additive) près.
A.2.3. Les équations (1) et (2) associées à l’équation d’état fournissent 5 équations scalaires comportant 6 champs scalaires inconnus (p, µ, T et les 3 composantes de la vitesse). On doit ajouter une équation décrivant l’évolution thermodynamique du fluide. (T=Cte ou S=Cte ou...)
A.2.4. L’entropie d’un système fermé de masse unité est conservée donc $\frac{{Ds}}{{Dt}} = \frac{{\partial s}}{{\partial t}} + \vec v.gr\vec ad\,s = 0$. (s étant un champ scalaire $(\vec v.gr\vec ad)s = \vec v.(gr\vec ad\,s)$ ). En multipliant l’équation par µ et en y ajoutant l’équation (1) multipliée par s on obtient ce qu’il faut avec ${\vec j_s} = \mu s\vec v$.
A.2.5. a. c a la dimension d’une vitesse. b. à l’ordre 1 $p' = \mu '{c^2}$ (3)
c. Dans l’équation (2’) on remplace $\vec v$ par $gr\vec ad\phi $ . On obtient $gr\vec ad\left( {{\mu _0}\frac{{\partial \phi }}{{\partial t}} + p'} \right) = \vec 0$ donc ${\mu _0}\frac{{\partial \phi }}{{\partial t}} + p'$ est une fonction de t uniquement. En utilisant l’indétermination mentionnée au A.2.2. on peut rendre nulle cette fonction. Alors $p' = - {\mu _0}\frac{{\partial \phi }}{{\partial t}}$ (4) et d’après (3) :
$\mu ' = - \frac{{{\mu _0}}}{{{c^2}}}\frac{{\partial \phi }}{{\partial t}}$ (5) En reportant ceci dans (1’) on constate alors que φ vérifie bien l’équation de d’Alembert. Par dérivation par rapport à t, il en est de même pour p’ et µ’.
A.2.6. a. Pour une onde plane (selon x’Ox) les champs ne sont des fonctions que de x et t.
b. La solution est la superposition des deux ondes progressives associées à f et g..
c. On pose $u = x - ct$ . Alors $\frac{\partial }{{\partial t}} \Leftrightarrow - c\frac{d}{{du}}$ et $\frac{\partial }{{\partial x}} \Leftrightarrow \frac{d}{{du}}$ et l’équation (2’) s’écrit $\frac{d}{{du}}\left( {p' - {\mu _0}cv} \right) = 0$ donc $p' = {\mu _0}cv$ (6) (la constante d’intégration est prise nulle si on veut que p’ soit nulle en valeur moyenne) On en déduit d’après (3) : $\mu ' = {\mu _0}\frac{v}{c}$ (7)
d. ${\left( {\frac{{\partial T}}{{\partial P}}} \right)_S} = - \frac{h}{{{c_p}}}$ (notations usuelles de la thermodynamique). Or $h = - T{\left( {\frac{{\partial V}}{{\partial T}}} \right)_P}$ c’est à dire (en utilisant une unité de masse $V = \frac{1}{\mu }$) $h = \frac{T}{{{\mu ^2}}}{\left( {\frac{{\partial \mu }}{{\partial T}}} \right)_P} = - \frac{T}{\mu }\beta $ . Alors ${\left( {\frac{{\partial T}}{{\partial P}}} \right)_S} = \frac{{T\beta }}{{\mu {c_p}}}$ et donc à l’ordre 1 $T' = \frac{{{T_0}\beta }}{{{\mu _0}{c_p}}}p' = \frac{{{T_0}\beta c}}{{{c_p}}}v$ (8)
e. $c = \sqrt {\gamma {{\left( {\frac{{\partial p}}{{\partial \mu }}} \right)}_T}} = \sqrt {\frac{{\gamma p}}{\mu }} = \sqrt {\frac{{\gamma RT}}{M}} = 347\;m.{s^{ - 1}}$
A.3.1. ${e_c} = \frac{v^2}{2}$
A.3.2. Pour une masse unité la variation de volume est $dV = d\left( {\frac{1}{\mu }} \right) = - \frac{1}{{{\mu ^2}}}d\mu \approx - \frac{1}{{\mu _0^2}}d\mu '$. Le travail reçu dû à la force de surpression est alors $ - p'dV = \frac{{p'}}{{\mu _0^2}}d\mu ' = \frac{{{c^2}}}{{\mu _0^2}}\mu 'd\mu '$ (en utilisant (3) ) ce qui correspond à l’augmentation de l’énergie potentielle (massique) ${e_{pot}} = \frac{{{c^2}}}{2}{\left( {\frac{{\mu '}}{{{\mu _0}}}} \right)^2}$
A.3.3. Grâce à (7) ${e_{pot}} = \frac{v^2}{2} = {e_c}$ donc $e = {e_c} + {e_{pot}} = {v^2}$ (9) (« équipartition de l’énergie »)
A.3.4. De façon générale, en multipliant l’équation (2’) par $\vec v$ et l’équation (1’) par p’/µ0 (qui vaut également $\frac{{\mu '}}{{{\mu _0}}}{c^2}$ d’après (3) ) et en faisant la somme on obtient :
$\vec v.gr\vec ad\,p' + p'div(\vec v) + {\mu _0}\vec v.\frac{{\partial \mathord{\buildrel{\lower3pt\hbox{$\scriptscriptstyle\leftarrow$}} \over v} }}{{\partial t}} + \frac{{{c^2}}}{{{\mu _0}}}\mu '\frac{{\partial \mu '}}{{\partial t}} = 0$ qui est bien l’équation proposée et qui est un bilan local d’énergie de densité volumique ${\mu _0}\left( {\frac{{{v^2}}}{2} + \frac{{{c^2}}}{2}{{\left( {\frac{{\mu '}}{{{\mu _0}}}} \right)}^2}} \right)$et de vecteur densité de courant $p'\vec v$. La vitesse de propagation de l’énergie est définie par analogie avec $\vec j = \rho \vec v$ selon ${v_{energie}} = \frac{{p'v}}{{{\mu _0}e}}$ et vaut (dans le cas d’une onde progressive seulement car on utilise (9) et (6) ) : ${v_{energie}} = \frac{{{\mu _0}cv\,v}}{{{\mu _0}\,{v^2}}} = c$
Partie B : propagation.
B.1.1. A la surface de séparation, on doit avoir continuité de la surpression et de la composante normale du champ de vitesse donc ${p_i}' + {p_r}' = {p_t}'$ et ${\vec v_i}.\vec n + {\vec v_r}.\vec n = {\vec v_t}.\vec n$ . Dans le cas d’une limite en x=0, la deuxième équation s’écrit : ${v_{i\,x}}(0,y,z,t) + {v_{r\,x}}(0,y,z,t) = {v_{t\,x}}(0,y,z,t)$.
B.1.2. Les conditions ci-dessus devant être vérifiées à tout instant et les fonctions exponentielles étant linéairement indépendantes, on en déduit que ${\omega _i} = {\omega _r} = {\omega _t}$.
B.1.3. De même, pour que les conditions soient vérifiées pour toutes les valeurs de y et z, il faut que les trois vecteurs d’onde aient même projection sur le plan x=0. Les lois de Descartes sont alors vérifiées sous la forme : $({\vec k_i},{\vec k_r},{\vec k_t},{\vec u_x})$ coplanaires et $\frac{{\sin {\theta _i}}}{{{c_1}}} = \frac{{\sin {\theta _r}}}{{{c_1}}} = \frac{{\sin {\theta _t}}}{{{c_2}}}$ (car ${k_i} = \frac{{{\omega _i}}}{{{c_i}}}$) . L’angle d’incidence limite vérifie alors : $\sin {\theta _0} = \frac{{{c_1}}}{{{c_2}}}$ (10)
B.1.4. En notation complexe $\vec v = i\vec k\phi \;\; \propto \;\frac{1}{c}A( \pm {\vec u_x})$ et d’après (4) $p'\;\; \propto {\mu _0}A$ . Les équations de continuité s’écrivent donc : $\left\{ \begin{array}{l}{\mu _1}\left( {{A_i} + {A_r}} \right) = {\mu _2}{A_t}\\\frac{1}{c_1}\left( {{A_i} - {A_r}} \right) = \frac{1}{c_2}{A_t}\end{array} \right.$ et alors $\left\{ \begin{array}{l}\frac{{{A_r}}}{{{A_i}}} = \frac{{{\mu _2}{c_2} - {\mu _1}{c_1}}}{{{\mu _2}{c_2} + {\mu _1}{c_1}}}\\\frac{{{A_t}}}{{{A_i}}} = \frac{{2{\mu _1}{c_2}}}{{{\mu _2}{c_2} + {\mu _1}{c_1}}}\end{array} \right.$ Comme $\Pi = p'v \propto \;\frac{{{\mu _0}}}{c}{A^2}$ on obtient $R{\rm{ = }}{\left( {\frac{{{\mu _{\rm{2}}}{c_2} - {\mu _{\rm{1}}}{c_1}}}{{{\mu _{\rm{2}}}{c_2} + {\mu _{\rm{1}}}{c_1}}}} \right)^2}\quad T = \frac{{4{\mu _1}\mu {}_2{c_1}{c_2}}}{{{{\left( {{\mu _{\rm{2}}}{c_2} + {\mu _{\rm{1}}}{c_1}} \right)}^2}}}$
R + T = 1 traduit ici la conservation de l’énergie
B.1.5. $\quad T = 1,26 1{0^{ - 3}}\quad R = {\rm{1}} - T \approx 1$
B.2. Remarque : en toute rigueur, l’impulsion sonore se propage à la vitesse de groupe que l’on identifie ici à la vitesse de phase dans l’approximation des ondes de faible amplitude.
B.2.1. a. Le trajet suivi par la lumière rend le chemin optique stationnaire par rapport aux trajets infiniment voisins.
b. Cela correspond à une durée stationnaire. (souvent extrémale voire minimale)
c. C ’est le principe de Fermat appliqué à l’acoustique.
B.2.2. a. Le trajet SIJM correspond à une arrivée en I et un départ de J avec un angle θ0 par rapport à la normale. Cela n’est possible que si L est assez grand.
b. $\Delta t = \frac{{\sqrt {{L^2} + {{\left( {h - h'} \right)}^2}} }}{{{c_1}}}$; $\Delta t' = \frac{{\sqrt {{L^2} + {{\left( {h + h'} \right)}^2}} }}{{{c_1}}}$; $\Delta t'' =\frac{L-\left( h+h' \right) tan\theta_0}{c_2}+\frac{h+h'}{c_{1}cos\theta_0}$
B.2.3. En utilisant (10) pour éliminer c2 on obtient ${{c}_{1}}\Delta t''=L\sin {{\theta }_{0}}+\left( h+h' \right)\cos {{\theta }_{0}}$ . On peut alors calculer ${{\left( {{c}_{1}}\Delta t' \right)}^{2}}-{{\left( {{c}_{1}}\Delta t'' \right)}^{2}}={{\left( L\cos {{\theta }_{0}}-\left( h+h' \right)\sin {{\theta }_{0}} \right)}^{2}}$ qui est positif donc $\Delta t'\ge \Delta t''$.(le cas d’égalité correspond à I=J)
B.2.4. En utilisant les valeurs numériques du B.1.5 on obtient : $\Delta t \approx \Delta t' = 0,294\;s$ $\Delta t''=0,083\ s$. Le trajet SIJM est de loin le plus court. En fait il n’est pas parcouru par l’onde car aux points I et J, le coefficient de transmission T est nul.
Partie C : absorption par conduction thermique.
C.1. a. Par conservation de l’énergie interne 2CTf = CT1 + CT2
b. $\Delta {S_U} = \Delta {S_1} + \Delta {S_2} = C\ln \left( {\frac{{{T_f}}}{{{T_1}}}} \right) + C\ln \left( {\frac{{{T_f}}}{{{T_2}}}} \right) = C\ln \left( {\frac{{{{\left( {{T_1} + {T_2}} \right)}^2}}}{{4{T_1}{T_2}}}} \right)$
c. A l’ordre 1 en $\frac{{\delta T}}{{{T_1}}} : \frac{{{{\left( {{T_1} + {T_2}} \right)}^2}}}{{4{T_1}{T_2}}} = \frac{{{{\left( {1 + \frac{{\delta T}}{{2{T_1}}}} \right)}^2}}}{{1 + \frac{{\delta T}}{{{T_1}}}}} \approx \frac{{1 + \frac{{\delta T}}{{{T_1}}}}}{{1 + \frac{{\delta T}}{{{T_1}}}}} = 1$ donc $\Delta {S_U} = 0$
C.2.1. La chaleur reçue par une masse donnée est $\iint{-{{{\vec{J}}}_{Q}}.d\vec{S}}\delta t=-\iiint{div({{{\vec{J}}}_{Q}})dV}\delta t$ Pour une masse élémentaire on écrira donc $\frac{{\delta Q}}{{\delta t}} = - div({\vec J_Q})dV = - div({\vec J_Q})\frac{{\delta m}}{\mu }$ Alors, en divisant par δm pour utiliser des grandeurs massiques, $\frac{{\delta {s_{ech}}}}{{\delta t}} = \frac{1}{T}\frac{{\delta q}}{{\delta t}} = - \frac{{div({{\vec J}_Q})}}{{\mu T}} = \frac{1}{{\mu T}}div(K\,gr\vec ad\,T))$ que l’on peut identifier à $\frac{{Ds}}{{Dt}}$ puisque le terme de création est d’ordre 2 en différence de températures alors que le terme d’échange que l’on vient d’évaluer est d’ordre 1.
C.2.2. a. La relation précédente sur $\frac{{Ds}}{{Dt}}$ s’écrit $\frac{{\partial s}}{{\partial t}} + \vec v.gr\vec ad(s) = \frac{1}{{\mu T}}div(K\,gr\vec ad\,T))$ En multipliant cette équation par µ et en ajoutant l’équation (1) multipliée par s on obtient $\frac{\partial }{{\partial t}}(\mu s) + div(\mu s\vec v) = \frac{1}{T}div(K\,gr\vec ad\,T))$ Le membre de droite peut s’écrire sous la forme $div(\frac{{K\,gr\vec ad\,T}}{T}) + \frac{K}{{{T^2}}}{\left( {gr\vec ad\,T} \right)^2}$ Finalement $div(\vec J{'_s}) + \frac{{\partial (\mu s)}}{{\partial t}} = \frac{K}{{{T^2}}}{\left( {gr\vec ad\,T} \right)^2} \ge 0$ . C’est l’équation locale de bilan d’entropie. Le membre de droite représente le taux volumique horaire de création d’entropie.
b. $\frac{\delta {{S}_{creation}}}{\delta t}=\iiint_{{{V}_{0}}}{\frac{K}{{{T}^{2}}}{{\left( gr\vec{a}d\,T \right)}^{2}}}dV$ qui est bien positif (second principe).
C.3. a. On calcule $gr\vec ad(T')$ en négligeant la dépendance de v0 par rapport à x. On voit alors apparaître dans le taux de création d’entropie un terme ${\left( {\sin (kx - \omega t)} \right)^2}$dont la valeur moyenne temporelle vaut $\frac{1}{2}$ donc $\left\langle {\frac{{\delta {S_{creation}}}}{{\delta t}}} \right\rangle = \frac{1}{2}K\Sigma ({x_2} - {x_1})\frac{{{\beta ^2}{c^2}}}{{c_p^2}}{\left[ {{v_0}(x)} \right]^2}{k^2}$ . Or l’énergie acoustique massique moyenne dans le volume est d’après (9) $\left\langle e \right\rangle = \frac{1}{2}{\left[ {{v_0}(x)} \right]^2}$ donc $\left\langle {\frac{{dE}}{{dt}}} \right\rangle = - K\Sigma ({x_2} - {x_1})\frac{{{T_0}{\beta ^2}{\omega ^2}}}{{c_p^2}}\left\langle e \right\rangle $
b. L’énergie acoustique moyenne entrant à l’abscisse x1 par unité de temps dans le volume étudié est $\Sigma \left\langle {{\Pi _x}} \right\rangle = \Sigma {\mu _0}c\left\langle {{{\left[ {v({x_1})} \right]}^2}} \right\rangle = \Sigma {\mu _0}c\left\langle {e(x{}_1)} \right\rangle $. La perte d’énergie doit être la différence entre ce qui entre en x1 et ce qui sort en x2 donc $ - \left\langle {\frac{{dE}}{{dt}}} \right\rangle = \Sigma {\mu _0}c\left( {\left\langle {e({x_1})} \right\rangle - \left\langle {e({x_2})} \right\rangle } \right)$ soit $\left\langle {\frac{{dE}}{{dt}}} \right\rangle \approx \Sigma ({x_2} - {x_1}){\mu _0}c\frac{{d\left\langle e \right\rangle }}{{dx}}$.
c. L’équation vérifiée par <e> est donc du type $\frac{{d\left\langle e \right\rangle }}{{dx}} = - 2\frac{{\left\langle e \right\rangle }}{\delta }$ donc $\left\langle e \right\rangle = {\left\langle e \right\rangle _0}\exp \left( { - 2\frac{x}{\delta }} \right)$ avec $\delta = 2\frac{{{\mu _0}c_p^2c}}{{K{T_0}{\beta ^2}{\omega ^2}}}$ ce qui est la relation de l’énoncé à condition de montrer que ${c_p} = \frac{{\beta {c^2}}}{{\gamma - 1}}$ ce qui se vérifie immédiatement pour un gaz parfait avec l’expression de c trouvée au A.2.6.e et le fait que $\beta = 1/T$.
d. En prenant c=340 ou 347 m.s-1, on obtient $\delta = 52\,{\rm{ou}}\,57\,km$ C’est énorme et peu compatible avec notre expérience de tous les jours (même si nous produisons des ondes sphériques plutôt que planes). Les ondes acoustiques sont atténuées avant cette distance pour d’autres raisons (viscosité essentiellement).
A 20 Hz, la limite de portée due à la viscosité est de 100 km mais elle n’est que de 10 m à 2000 Hz. Les éléphants ont un moyen de communication bien efficace !
Partie D : ondes acoustiques de grande amplitude.
D.1.1. Pour une onde plane (1) devient $\frac{\partial }{{\partial x}}(\mu v) + \frac{{\partial \mu }}{{\partial t}} = 0$ et (2) devient $\mu \frac{{\partial v}}{{\partial t}} + \mu v\frac{{\partial v}}{{\partial x}} = - \frac{{\partial p}}{{\partial x}}$ .
D.1.2. $\begin{array}{l}\frac{{\partial p}}{{\partial x}}\quad \frac{d}{{dp}}\left( {\mu v} \right)\quad + \quad \frac{{\partial p}}{{\partial t}}\quad \frac{d}{{dp}}\left( \mu \right)\quad = \quad 0\\\frac{{\partial p}}{{\partial x}}\left( {1 + \mu v\frac{{dv}}{{dp}}} \right)\,\, + \quad \frac{{\partial p}}{{\partial t}}\quad \mu \frac{{dv}}{{dp}}\quad \;\; = \quad 0\end{array}$ (11)
D.1.3. Le système (11) admet des solutions non nulles si son déterminant est nul donc si ${\left( {\mu \frac{{dv}}{{dp}}} \right)^2} - \frac{{d\mu }}{{dp}} = 0$ c.q.f.d. puisque (à entropie constante) $\frac{{d\mu }}{{dp}} = {\left( {\frac{{\partial \mu }}{{\partial p}}} \right)_S} = \frac{1}{{{c^2}}}$ . (11) se réduit alors à sa seconde équation qui devient (signe « + ») $\frac{{\partial p}}{{\partial x}}\left( {1 + \frac{v}{c}} \right) + \frac{1}{c}\frac{{\partial p}}{{\partial t}} = 0$ (12)
D.1.4. L’équation (12) contient (à un facteur c près) une dérivée particulaire de p associée à un « déplacement » à la vitesse c+v. Le champ de pression se propage donc à la vitesse c+v . Or c (d’après l’énoncé) et v (car on a choisi le signe « + ») sont des fonctions croissantes de p. Dans un front de surpression qui se propage, les fortes pressions vont rattraper les faibles, le front va se raidir jusqu’à une pente infinie : discontinuité de pression.
D.2. Le système fermé étudié, de masse δm est compris entre les traits pointillés. A l’instant t, il est dans l’état de repos 2. A l’instant t+δt, il est dans l’état perturbé 1. On raisonne sur une section droite Σ0 prise comme unité. On pose donc Σ0 = 1.
D.2.1. Conservation de la masse :
$\delta m = {\mu _0}c'\delta t = \left( {{\mu _0} + \mu '} \right)(c' - V)\delta t$ $\mu ' = \frac{{{\mu _0}V}}{{c' - V}}$ (13)
D.2.2. Bilan de quantité de mouvement : la force horizontale totale est p1 - p2 = p’
$p'\delta t = {P_{finale}} = \delta mV = {\mu _0}c'\delta tV$ soit $p' = {\mu _0}c'V$ (14)
(à comparer avec l’équation (6) )
D.2.3. $u = \frac{P}{{\mu (\gamma - 1)}}$ Seule la pression p1 travaille. Elle contribue à la variation de l’énergie interne et de l’énergie cinétique macroscopique du gaz :
$({p_0} + p')V\delta t = \frac{1}{2}\delta m{V^2} + \frac{{\delta m}}{{\gamma - 1}}\left( {\frac{{{p_0} + p'}}{{{\mu _0} + \mu '}}} \right) - \frac{{\delta m}}{{\gamma - 1}}\left( {\frac{{{p_0}}}{{{\mu _0}}}} \right)$ $ \Rightarrow $$({p_0} + p')V = \frac{1}{2}{\mu _0}c'{V^2} + \frac{{(c' - V)}}{{\gamma - 1}}\left( {{p_0} + p'} \right) - \frac{{c'}}{{\gamma - 1}}\left( {{p_0}} \right)$
soit $\frac{\gamma }{{\gamma - 1}}\left( {{p_0} + p'} \right)V - \frac{{p'c'}}{{\gamma - 1}} = \frac{1}{2}{\mu _0}c'{V^2}$ (15)
D.2.4. On élimine c’ en l’exprimant en fonction de p’ grâce à (14) pour le reporter dans (15) et obtenir (en notant X le rapport p’/p0) : ${X^2} - X\frac{{\gamma \left( {\gamma + 1} \right)}}{2}\frac{{{V^2}}}{{c_0^2}} - {\gamma ^2}\frac{{{V^2}}}{{c_0^2}} = 0$ qui se résoud en : $\frac{{p'}}{{{p_0}}} = \frac{{\gamma \left( {\gamma + 1} \right)}}{4}\frac{{{V^2}}}{{c_0^2}} + \sqrt {{\gamma ^2}\frac{{{V^2}}}{{c_0^2}} + {{\left( {\frac{{\gamma \left( {\gamma + 1} \right)}}{4}} \right)}^2}{{\left( {\frac{V}{{{c_0}}}} \right)}^4}} $ puis $\frac{{c'}}{{{c_0}}} = \frac{{\left( {\gamma + 1} \right)}}{4}\frac{V}{{{c_0}}} + \sqrt {1 + {{\left( {\frac{{\gamma + 1}}{4}} \right)}^2}{{\left( {\frac{V}{{{c_0}}}} \right)}^2}} $
Remarque : pour V << c0 on obtient $c' \approx {c_0}$
D.2.5. a. $s = \frac{R}{{M\left( {\gamma - 1} \right)}}\ln \left( {\frac{{p\mu _0^\gamma }}{{{p_0}{\mu ^\gamma }}}} \right)$ b. $\frac{{\delta {S_{creation}}}}{{{\Sigma _0}\delta t}} = \frac{{R{\mu _0}c'}}{{M(\gamma - 1)}}\ln \left( {\frac{{1 + \frac{{p'}}{{{p_0}}}}}{{{{\left( {1 + \frac{{\mu '}}{{{\mu _0}}}} \right)}^\gamma }}}} \right)$
c. $\frac{{c'}}{{{c_0}}} = 1,34$ $\frac{{p'}}{{{p_0}}} = 0,94$ $\frac{{{\mu _1}}}{{{\mu _0}}} = 1,59$ $\frac{{\delta {S_{creation}}}}{{{\Sigma _0}\delta t}} \approx 5000\,J{K^{ - 1}}{s^{ - 1}}{m^{ - 2}}$ (c’=340 m.s-1)
La création d’entropie est due à l’irréversibilité de l’onde de choc (déséquilibre mécanique).

Concours Physique ENS Cachan 1995 (Corrigé)

Avertissement:
1) les graphes étant tous semblables (des exponentielles croissantes), il n’en a été tracé que l’allure;
2) dans la question 3.3.3-, mes applications numériques me conduisent toujours à un régime oscillant, qui n’est pas dans la logique de l’énoncé; j’ai donc renoncé au tracé demandé.
ENS Cachan B(A) 1995
Première partie: Étude électromagnétique
1.1- Spire équivalente:
1.1.1- La force de Laplace a pour expresion: ${\rm{d\vec f}} = {\rm{dI}}{\kern 1pt} {\rm{\vec u}} \wedge {\rm{\vec B}}$
1.1.2- On considère une spire élémentaire de section ldr, de longueur 2πr, parcourue par le courant d’intensité ${\rm{dI}} = \delta {\rm{ldr}}$, où $\delta =\frac{I}{\left[ l \left( R_e - R_i \right) \right]}$; elle est soumise à la force:
${\rm{\vec f}} = \int_{{{\rm{R}}_{\rm{i}}}}^{{\rm{Re}}} {\;\delta {\rm{ldr2}}\pi {\rm{rB}}{{{\rm{\vec e}}}_{\rm{z}}}} = \delta \frac{{\rm{L}}}{{\rm{N}}}\pi {\rm{B}}\left( {{{\rm{R}}_{\rm{e}}}^2 - {{\rm{R}}_{\rm{i}}}^2} \right){{\rm{\vec e}}_{\rm{z}}}$
1.1.3- On constate en effet que l’on peut écrire:
${\rm{\vec f}} = {\rm{IB2}}\pi {\rm{R}}{{\rm{\vec e}}_{\rm{z}}}$
1.2- Force électromagnétique résultante:
1.2.1- Le résultat est immédiat: ${\rm{\vec F}} = {\rm{N\vec f}} = {\rm{NIB2}}\pi {\rm{R}}{{\rm{\vec e}}_{\rm{z}}}$
1.2.2- On peut écrire F sous la forme indiquée en posant: ${{\rm{k}}_\phi } = {\rm{B2}}\pi {\rm{R}}$
C’est homogène à une circulation du champ magnétique B, mais cela ne correspond pas à une circulation concrête.
1.2.3- AN: ${{\rm{I}}_{\rm{e}}} = \delta {\rm{L}}\left( {{{\rm{R}}_{\rm{e}}} - {{\rm{R}}_{\rm{i}}}} \right) = 800\;{\rm{A}}\,{\rm{;}}\quad {\rm{F}} = {\rm{78}}{\rm{,4}}\;{\rm{N}}$
1.3- Force contre électromotrice:
1.3.1- On sait que le champ électromoteur est: ${{\rm{\vec E}}_{\rm{M}}} = {\rm{\vec v}} \wedge {\rm{\vec B}} = {\rm{Bv}}{{\rm{\vec u}}_\theta }$ et la fem: ${\rm{e}} = \int {{{{\rm{\vec E}}}_{\rm{M}}}.{\rm{d\vec M}}} = {\rm{Bvu}}$
1.3.2- Il est clair que: ${\rm{e}} = {\rm{Bv2}}\pi {\rm{R}}\quad \Rightarrow \quad {{\rm{k}}^{\rm{'}}}_\phi = {\rm{B2}}\pi {\rm{R}} = {{\rm{k}}_\phi }$ AN: ${{\rm{e}}_{\rm{0}}} = {\rm{9}}{\rm{,80}}{\rm{.1}}{{\rm{0}}^{{\rm{ - 2}}}}\;{\rm{V}}$
1.3.3- Le résultat est immédiat: ${\rm{E}} = {\rm{Ne}}$
1.4- Calcul de la résistance:
1.4.1- La résistance se calcule par la formule:
${\rm{R}} = \rho \frac{{\rm{l}}}{{\rm{S}}}\quad \Rightarrow \quad {{\rm{R}}_{\rm{b}}} = \rho \frac{{{\rm{2}}\pi {\rm{R}}{{\rm{N}}^{\rm{2}}}}}{{{\rm{L}}\left( {{{\rm{R}}_{\rm{e}}} - {{\rm{R}}_{\rm{i}}}} \right)}}\quad \Rightarrow \quad $${{\rm{r}}_{\rm{b}}} = \rho \frac{{{\rm{2}}\pi {\rm{R}}}}{{{\rm{L}}\left( {{{\rm{R}}_{\rm{e}}} - {{\rm{R}}_{\rm{i}}}} \right)}}$
1.4.2- AN: ${{\rm{r}}_{\rm{b}}} = {\rm{1}}{\rm{,23}}{\rm{.1}}{{\rm{0}}^{{\rm{ - 4}}}}\;\Omega $
1.5- Calcul de l’inductance:
1.5.1.a- Tout plan contenant l’axe Oz est plan d’antisymétrie pour la distribution de courants; il s’ensuit que le champ magnétique${\rm{\vec B}}$ est dirigé selon l’axe Oz, la « règle du tire-bouchon » donnant alors le sens du vecteur.
À partir de la loi de Biot & Savart, on retrouve le résultat:
${\rm{dB = }}\frac{{{\mu _{\rm{0}}}}}{{{\rm{2R}}}}\frac{{{{\rm{I}}_{\rm{e}}}{\rm{dZ}}}}{{\rm{L}}}{\rm{si}}{{\rm{n}}^{\rm{3}}}\left( \alpha \right)$
où Z est la position de la spire (z étant celle du point M).
1.5.1.b- Dans un premier temps, on a intérêt à intégrer selon la variable α:
${\rm{cot}}\left( \alpha \right) = \frac{{{\rm{z}} - {\rm{Z}}}}{{\rm{R}}}\quad \Rightarrow \quad {\rm{dZ}} = \frac{{{\rm{Rd}}\alpha }}{{{\rm{si}}{{\rm{n}}^{\rm{2}}}\left( \alpha \right)}}\quad \Rightarrow \quad {\rm{dB}} = \frac{{{\mu _0}{{\rm{I}}_{\rm{e}}}}}{{2{\rm{L}}}}{\rm{sin}}\left( \alpha \right){\rm{d}}\alpha \quad \Rightarrow \quad {\rm{B}} = \frac{{{\mu _{\rm{0}}}{{\rm{I}}_{\rm{e}}}}}{{{\rm{2L}}}}\left[ {{\rm{cos}}\left( {{\alpha _{\rm{1}}}} \right) - {\rm{cos}}\left( {{\alpha _{\rm{2}}}} \right)} \right]$
On en déduit: ${\rm{\vec B}}\left( {\rm{z}} \right) = \frac{{{\mu _{\rm{0}}}{{\rm{I}}_{\rm{e}}}}}{{{\rm{2L}}}}\left[ {\frac{{\rm{z}}}{{\sqrt {{{\rm{R}}^{\rm{2}}} + {{\rm{z}}^{\rm{2}}}} }} + \frac{{{\rm{L}} - {\rm{z}}}}{{\sqrt {{{\rm{R}}^{\rm{2}}} + {{\left( {{\rm{L}} - {\rm{z}}} \right)}^2}} }}} \right]{{\rm{\vec e}}_{\rm{z}}}$
1.5.1.c- On calcule la valeur du rapport demandé et l’on constate que B varie relativement peu sur l’axe, à l’intérieur de la spire.
$B \left( 0 \right) = \frac{\mu_{0} I_{e}}{2 \sqrt{R{2} + L^{2}}} \ ; \ B \left( \frac{L}{2} \right) = \frac{\mu _{0}I_{e}}{\sqrt {4R^{2} + L^{2}}} \Rightarrow  \frac{B \left( 0 \right)}{B \left( \frac{L}{2} \right)}= \frac{\sqrt {4R^{2} + L^{2}}}{2 \sqrt{R^{2} + L^{2}}} = 0,785$
1.5.2- Compte tenu des hypothèses simplificatrices, le calcul est aisé:
${{\text{W}}_{\text{mag}}}=\iiint{\frac{{{\text{B}}^{\text{2}}}}{\text{2}{{\mu }_{\text{0}}}}\,}\text{d}\tau =\frac{{{\text{B}}^{\text{2}}}}{\text{2}{{\mu }_{\text{0}}}}\pi {{\text{R}}^{\text{2}}}\text{L}\quad \Rightarrow \quad {{\rm{W}}_{{\rm{mag}}}} = \frac{{\pi {\mu _{\rm{0}}}{{\rm{R}}^{\rm{2}}}{\rm{L}}{{\rm{I}}_{\rm{e}}}^2}}{{{\rm{2}}\left( {{\rm{4}}{{\rm{R}}^{\rm{2}}} + {{\rm{L}}^{\rm{2}}}} \right)}}$
1.5.3- De la relation: ${{\text{W}}_{\text{mag}}}=\frac{1}{2}{{\text{I}}^{2}}$, on tire:
$=\frac{\pi {{\mu }_{0}}{{\text{R}}^{\text{2}}}\text{L}}{\text{4}{{\text{R}}^{\text{2}}}+{{\text{L}}^{\text{2}}}}\,{{\text{N}}^{\text{2}}}$ $=\frac{\pi {{\mu }_{0}}{{\text{R}}^{\text{2}}}\text{L}}{\text{4}{{\text{R}}^{\text{2}}}+{{\text{L}}^{\text{2}}}}={{1,56.10}^{-8}}\ H$
1.6- Équation différentielle:
1.6.1- On a affaire à un circuit série comportant un générateur de fem U, une force électromotrice E, une inductance propre L et une résistance Rb . L’équation différentielle du circuit est:
$\text{U}=\text{E+}\frac{\text{dI}}{\text{dt}}+{{R}_{b}}I$
1.6.2- En remplaçant les grandeurs par leurs expressions:
$\text{U}=\text{N}\left( {{\text{k}}_{\phi }}v+\frac{d{{I}_{e}}}{dt}+{{r}_{b}}{{I}_{e}} \right)$
1.7- Détermination du nombre de spires:
1.7.1- Des deux équations: ${U_0} = {N_0}\left( {{k_\phi }{V_0} + {r_b}{I_e}} \right)$ et $F = {k_\phi }{I_e}$, on déduit: ${N_0} = \frac{U_0}{k_{\phi }V_{0} + r_{b}\frac{F}{k_{\phi}}}$
1.7.2- ${N_0}$ permet d’ajuster la valeur de $k_{\phi }V_{0} + r_{b}\frac{F}{k_\phi }$.
1.7.3- AN: ${N_0} = 61\;spires$ ${R_b} = 0,456\,\Omega $ $=58,2\,\mu H$ ${E_0} = 5,98\,V$
1.8- Échelon de tension:
1.8.1- cas a: solénoïde immobilisé; pour $t = 0$, la forme de l’équation différentielle imposant la continuité de l’intensité, elle s’intègre en:
$\frac{di}{dt}+{{R}_{b}}i={{U}_{0}}\quad \Rightarrow \quad $${{i}_{a}}\left( t \right)=\frac{{{U}_{0}}}{{{R}_{b}}}\left[ 1-\exp \left( -\frac{{{R}_{b}}}{}t \right) \right]$
1.8.1- cas b: solénoïde à vitesse uniforme; il suffit de changer ${U_0}$ en ${U_0} - {E_0}$, soit:
$\frac{di}{dt}+{{R}_{b}}i={{U}_{0}}-{{E}_{0}}\quad \Rightarrow \quad $${{i}_{b}}\left( t \right)=\frac{{{U}_{0}}-{{E}_{0}}}{{{R}_{b}}}\left[ 1-\exp \left( -\frac{{{R}_{b}}}{}t \right) \right]$
AN: ${i_a}{\left( t \right)_{\left[ A \right]}} = 27,2\exp \left( { - {{7,84.10}^3}t} \right)$
${i_b}{\left( t \right)_{\left[ A \right]}} = 13,9\exp \left( { - {{7,84.10}^3}t} \right)$
Les exponentielles appartenant à la catégorie des « fonctions bien connues », seule l’allure des courbes a été tracée.
1.8.2- La constante de temps ne dépend pas du nombre de spires car elle vaut:
${{\tau }_{e}}=\frac{}{{{\text{R}}_{\text{b}}}}=\frac{{{\mu }_{0}}{{L}^{2}}\left( {{R}_{e}}-{{R}_{i}} \right)R}{2\rho \,\left( \text{4}{{\text{R}}^{\text{2}}}+{{\text{L}}^{\text{2}}} \right)}=128\,\mu s$
Deuxième partie: Étude thermique
2.1- Pertes Joule:
La puissance dissipée par effet Joule est égale à:
${P_J} = {R_b}{I^2} = \rho \frac{{{\rm{2}}\pi {\rm{R}}{{\rm{N}}^{\rm{2}}}}}{{{\rm{L}}\left( {{{\rm{R}}_{\rm{e}}} - {{\rm{R}}_{\rm{i}}}} \right)}}{\left[ {\delta \frac{L}{N}\left( {{{\rm{R}}_{\rm{e}}} - {{\rm{R}}_{\rm{i}}}} \right)} \right]^2}\quad \Rightarrow \quad $${P_J} = \pi \rho {\delta ^2}=L\left( {{R_e}^2 - {R_i}^2} \right) = 78,4\;W$
La densité volumique de puissance peut s’écrire directement, sans passer par ${P_J}$:
${p_J} = \rho {\delta ^2} = 32,0 \ MW/m^3$
2.2- Schéma équivalent:
2.2.1- Le schéma équivalent correspond à l’équation: ${P_J} = {C_{th}}\frac{{d\left( {\Delta \theta } \right)}}{{dt}} + \frac{{\Delta \theta }}{{{R_{th}}}}$.
La capacité thermique (et non calorifique) du solénoïde vaut: ${C_{th}} = c\varpi \pi \left( {{R_e}^2 - {R_i}^2} \right)$
Pour obtenir la résistance thermique d’échange, on exprime cet échange de deux façons:
$\frac{{\Delta \theta }}{{{R_{th}}}} = \alpha 2\pi {R_i}L\Delta \theta + \alpha 2\pi {R_e}L\Delta \theta \quad \Rightarrow \quad $${R_{th}} = \frac{1}{{\alpha 2\pi L\left( {{R_i} + {R_e}} \right)}}$
2.2.2- AN: ${C_{th}} = 425\;J.{K^{ - 1}}$ ${R_{th}} = 17,0\;K.{W^{ - 1}}$
2.2.3- Les échanges de chaleur se font par conduction, convexion et rayonnement.
2.3- Évolution de la température du solénoïde:
2.3.1- L’équation différentielle du 2.2.1- s’intègre comme celle du 1.8.1-:
${\theta _S} = {\theta _{amb}} + {P_J}{R_{th}}\left[ {1 - \exp \left( { - \frac{t}{{{R_{th}}{C_{th}}}}} \right)} \right]$
$\theta {}_S = 20 + 1333\left[ {1 - \exp \left( { - {{1,384.10}^{ - 4}}t} \right)} \right]$
À nouveau, seule l’allure du graphe est donnée.
2.3.2- La constante de temps vaut: ${\tau _{th}} = {R_{th}}{C_{th}} = 7,23\;ks$
2.3.3- Le calcul de la densité de courant est immédiat: ${P_J}{R_{th}} = 100\;K\quad \Rightarrow \quad $${\delta _P} = 11,0\;A.m{m^{ - 2}}$
2.3.4- La durée maximale est: ${t_{\max }} = - {\tau _{th}}\ln \left( {1 - \frac{{{\theta _{S\max }} - {\theta _{amb}}}}{{P{}_J{R_{th}}}}} \right) = 564\;s$
2.3.5.a- Physiquement, cela veut dire que la chaleur produite est restée dans la spire et n’a pas encore commencé à être évacuée vers l’extérieur; l’équation du 2.2.1- se résume à:
${P_J} \approx {C_{th}}\frac{{d\left( {\Delta \theta } \right)}}{{dt}}\quad \Rightarrow \quad $$\Delta \theta \left( t \right) \approx \frac{{{P_J}}}{{{C_{th}}}}t = \frac{{\rho L{\delta ^2}}}{{c\varpi }}t$ $\quad \Rightarrow \quad $$\Delta \theta \left( {{T_0}} \right) \approx 1,84\;^\circ C$
Mathématiquement, dans l’équation du 2.3.1-, on développe:$\exp \left( \varepsilon \right) \approx 1 + \varepsilon $.
2.3.5.b- On déduit de ce qui précède la densité maximale de courant: ${\delta _{\max }} \approx \sqrt {\frac{{c\varpi \left( {{\theta _{S\max }} - {\theta _{amb}}} \right)}}{{\rho L{T_0}}}} $
2.3.5.c- AN: ${\left( {{\delta _{\max }}} \right)_{Cu}} \approx 294\;A.m{m^{ - 2}}$ ${\left( {{\delta _{\max }}} \right)_{Al}} \approx 198\;A.m{m^{ - 2}}$
2.3.5.d- On calcule F à partir de la formule:
$F = \pi BL\delta \left( {{R_e}^2 - {R_i}^2} \right)$ $\quad \Rightarrow \quad $${F_{Cu}} = 576\;N$ $F{}_{Al} = 388\;N$
2.3.5.e- En régime permanent, ${P_J}{R_{th}} = \frac{\rho }{{2\alpha }}\left( {{R_e} - {R_i}} \right){\delta ^2} = \Delta \theta $, dont on déduit:
$\left\{ \begin{array}{l}{\delta _{Cu}} = 11,0\;A.m{m^{ - 2}}\; \Rightarrow \;{F_{Cu}} = 21,5\;N\\{\delta _{Al}} = 8,80\;A.m{m^{ - 2}}\; \Rightarrow \;{F_{Al}} = 17,2\;N\end{array} \right.$
Troisième partie: Étude électromécanique et thermique
3.1- Masse du solénoïde:
La masse du solénoïde est égale à: $M = \varpi \pi L\left( {{R_e}^2 - {R_i}^2} \right)\,;\quad {M_{Cu}} = 21,8\;g\,;\quad {M_{Al}} = 6,62\;g$
3.2- Solénoïde ouvert:
L’équation différentielle $M\frac{{dv}}{{dt}} + \mu v = F$ s’intègre en:
$v = \frac{{{F_0}}}{\mu }\left[ {1 - \exp \left( { - \frac{\mu }{M}t} \right)} \right]$
La constante de temps mécanique a pour valeur:
${\left( {{\tau _m}} \right)_{Cu}} = \frac{M}{\mu } = 0,218 \ s$
3.3- Solénoïde alimenté:
3.3.1- Les équations rappelées par l’énoncé sont les suivantes:
1.2.2-$\;{\rm{F = }}{{\rm{k}}_\phi }NI\;;$ 1.3.3- $E = N{k_\phi }v\;;$ 1.6.1- $U=E+\frac{dI}{dt}+{{R}_{b}}I$
Par élimination de I entre les équations: $M\frac{{dv}}{{dt}} + \mu v = {k_\phi }NI$, et: $U=N{{k}_{\phi }}v+\frac{dI}{dt}+{{R}_{b}}I$, on trouve:
$U=\frac{M}{{{k}_{\phi }}N}\frac{{{d}^{2}}v}{d{{t}^{2}}}+\frac{\mu +M{{R}_{b}}}{{{k}_{\phi }}N}\frac{dv}{dt}+\left( {{k}_{\phi }}N+\frac{{{R}_{b}}\mu }{{{k}_{\phi }}N} \right)v\ ;\ a=\frac{M}{{{k}_{\phi }}N}\ ;\ b=\frac{\mu +M{{R}_{b}}}{{{k}_{\phi }}N}\ ;\ c={{k}_{\phi }}N+\frac{{{R}_{b}}\mu }{{{k}_{\phi }}N}$
3.3.2- La première approximation: ${{\tau }_{e}}=\frac{}{{{R}_{b}}}<<{{\tau }_{m}}=\frac{M}{\mu }$ s’écrit: $\mu <<{{R}_{b}}M\quad \Rightarrow \quad b\approx \frac{M{{R}_{b}}}{{{k}_{\phi }}N}$.
La deuxième approximation conduit à: $c \approx {k_\phi }N$.
Une solution particulière est: $v = \frac{U_0}{c}$.
On cherche la solution générale de l’équation sans second membre sous la forme: $v = V\exp \left( { - \frac{t}{\tau }} \right)$; on aboutit à l’équation caractéristique: $c{\tau ^2} - b\tau + a = 0$, dont on supposera les racines réelles positives:
$\Delta = b{}^2 - 4ca \ge 0$;${\tau _1}{\tau _2} = \frac{a}{c} > 0$ et :${\tau _1} + {\tau _2} = \frac{b}{c} > 0$.
Il vient alors: ${\tau _1} = \frac{{b - \sqrt {{b^2} - 4ca} }}{{2c}}\;;\quad {\tau _2} = \frac{{b + \sqrt {{b^2} - 4ca} }}{{2c}}$
À partir de la solution: $v\left( t \right) = {V_A}\exp \left( { - \frac{t}{{{\tau _1}}}} \right) + {V_B}\exp \left( { - \frac{t}{{{\tau _2}}}} \right) + {V_C}$, les conditions initiales donnent:
$v\left( {t = 0} \right) = {V_A} + {V_B} + \frac{{{U_0}}}{c}\;;\quad {\left( {\frac{{dv}}{{dt}}} \right)_{t = 0}} = 0 = - \frac{{{V_A}}}{{{\tau _1}}} - \frac{{{V_B}}}{{{\tau _2}}}$
dont on tire: ${V_A} = \frac{{{U_0}}}{c}\frac{{{\tau _1}}}{{{\tau _2} - {\tau _1}}}\;;\quad {V_B} = - \frac{{{U_0}}}{c}\frac{{{\tau _2}}}{{{\tau _2} - {\tau _1}}}$, soit:
${{\tau }_{1}}=\frac{M{{R}_{b}}-\sqrt{{{\left( M{{R}_{b}} \right)}^{2}}-4M{{\left( {{k}_{\phi }}N \right)}^{2}}}}{2{{\left( {{k}_{\phi }}N \right)}^{2}}}\ ;\ {{\tau }_{2}}=\frac{M{{R}_{b}}+\sqrt{{{\left( M{{R}_{b}} \right)}^{2}}-4M{{\left( {{k}_{\phi }}N \right)}^{2}}}}{2{{\left( {{k}_{\phi }}N \right)}^{2}}}\ ;\ {{V}_{C}}=\frac{{{U}_{0}}}{{{k}_{\phi }}N}$
${{V}_{A}}=\frac{{{U}_{0}}}{{{k}_{\phi }}N}\frac{M{{R}_{b}}-\sqrt{{{\left( M{{R}_{b}} \right)}^{2}}-4M{{\left( {{k}_{\phi }}N \right)}^{2}}}}{2\sqrt{{{\left( M{{R}_{b}} \right)}^{2}}-4M{{\left( {{k}_{\phi }}N \right)}^{2}}}}\ ;\ {{V}_{B}}=-\frac{{{U}_{0}}}{{{k}_{\phi }}N}\frac{M{{R}_{b}}+\sqrt{{{\left( M{{R}_{b}} \right)}^{2}}-4M{{\left( {{k}_{\phi }}N \right)}^{2}}}}{2\sqrt{{{\left( M{{R}_{b}} \right)}^{2}}-4M{{\left( {{k}_{\phi }}N \right)}^{2}}}}$
L’hypothèse ${\tau _2} > > {\tau _1}$ implique ${V_A} < < {V_B}$ et $\exp \left( { - \frac{t}{{{\tau _1}}}} \right) < < \exp \left( { - \frac{t}{{{\tau _2}}}} \right)$; par suite:
$v\left( t \right) \approx \frac{{{U_0}}}{{{k_\phi }N}}\left[ {1 - \exp \left( { - \frac{t}{{{\tau _2}}}} \right)} \right]$
3.3.3- Plusieurs essais me conduisent toujours à un $\Delta $négatif, donc à un ${\tau _2}$ complexe, ce qui ne correspond pas à l’esprit dans lequel a été rédigé l’énoncé.
3.4- Analyse finale:
3.4.1- Les constantes de temps ont des ordres de grandeur très différents:
${\tau _e} = 128\;\mu s\quad < < \quad {\tau _m} = 0,218\;s\quad < < \quad {\tau _{th}} = 7,23\;ks$
3.4.2- La question 3.2- conduit au résultat: $a = \frac{{dv}}{{dt}} = \frac{{{F_0}}}{M}$
3.4.3- On calcule donc: ${a_{\max }} = \frac{{{F_{\max }}}}{M} = \left\{ \begin{array}{l}26,4\;mm.{s^{ - 2}}\;pour\;Cu\\58,6\;mm.{s^{ - 2}}\;pour\;Al\end{array} \right.$
3.4.4- De la question 2.3.5.e-, on tire: ${a_{\max C}} = \left\{ \begin{array}{l}0,986\;mm.{s^{ - 2}}\;pour\;Cu\\2,60\;mm.{s^{ - 2}}\;pour\;Al\end{array} \right.$
3.4.5- On constate que c’est l’aluminium qui s’avère le plus intéressant, contrairement à l’idée que l’on a couramment.

Autres Concours

2011  : Concours ENAC de  physique 2011  :  énoncé ,  corrigé Concours ICNA de  physique 2011  :  énoncé ,  corrigé Concours ICNA de ...